Sunteți pe pagina 1din 97

Psychiatry

.


Psychiatry Final Questions 2010-2012
1) A patient with hallucinations & extreme agitation, his urine screen was
positive for phenylcyclidine (PCP). Which is best for his severe agitation?
1
2
3
4

+ Haloperidol
Citalopram
Fluoxetine
Triptyline
-- PCP is stimulant. only haloperidol is antipsychotic here

2) A patient had commanded suicide hallucinations. He was treated with high


dose haloperidol then developed torticollis. The best treatment is:
1 +Kemadrin (procyclidine)
2 Low dose neuroleptics
3 Acetaminophen
-- muscle relaxant
3) Echolalia, echopraxia and catalepsy are signs of which subtype of
schizophrenia?
1
2
3

4
5

Undifferentiated
+Catatonic
Paranoid
Disorganized
Residual
-- has been dropped from DSV 5

4) If a patient had schizophrenia, risk of his brother to develop


schizophrenia is:
1 +10%

2 5%
3 20%
4 30%
-- risk for FDR to have schizophrenia is 10% same with
monozygotic twin
5) 25 years old male was in good health. Now he complains of anhedonia; his
mother died since 1 month, so now he is extremely sad & tearful. He lost 3 lb
and he can't sleep at night. Most likely the patient developed:
1
2
3
4
5

Adjustment disorder
PTSD
+Uncomplicated bereavement
Major depression
Dysthymic disorder
-- duration of one month is important to diagnose

6) Parents of a 4-year-old girl presented her because they think she has
abnormal development. She developed normally until 2-3 years, she spoke
words & sentences, & used to play with her mother. Now since 3 months, she
lost what she developed, doesnt speak at all, doesnt play, & lost bowel
continence after which she was free of diaper almost 1 year. She is most
likely to have:
1
2
3
4
5

+Child disintegrative disorder


Autism
Retts disease
Asperger's disease
Pervasive developmental disorder
-- Childhood disintegrative disorder, also known as Heller's
syndrome is a very rare condition in which children develop
normally until at least two years of age, but then demonstrate
a severe loss of social, communication and other skills.

7) An 8 years old child becomes very distressed & tearful every morning
before school. She complains of abdominal pain, and if she goes to school,

she continues complaining so that her mother goes many times and takes her
home early. She takes many days off school with same complaints. Her signs
disappear on weekends unless her parents went out and left her with a
babysitter. The child mostly has:
1+Separation anxiety disorder
2Attachment disorder
3Generalized anxiety
-- attachment disorder definition: child has impaired social
interactions that are due to pathologic parenting and not due to
an autism spectrum disorder.
8) A middle aged man, preoccupied with his health, is always concerned that
his abnormal bowel movement indicates cancer. Recently he is worried about
having MI although his doctor gives him reassurance & his tests are normal
all the time. The patient mostly has:
1 +Hypochondriasis
2 Conversion
3 Somatization
-- hypochondriasis is excessive worrying of a specific
disease/diagnosis
9) 19 years old, college student, had urinary incontinence which improved
slowly over few weeks. Now she presented with blurred vision in her right
eye after breakup with her boyfriend. She mostly has:
1 +Somatoform disorder
2 Multiple sclerosis
-- recurring, multiple, clinically significant physical complaints
which result in patient seeking treatment or having impaired
functioning

10) A female was referred to a psychiatric clinic after she called the police
to take her because she was responsible for her neighbor abortion. She said
that she had thoughts about her neighbor having abortion, and she feels
threatened from her neighbor because she thinks that her thoughts are
heard across the walls. The patient believes that her thoughts of her
neighbor abortion could make it happen. This is called:
1
2
3
4
5

Magical thinking
Displacement
Projection
Ideas of reference
+Thought broadcasting
-- She believes others can read her thought

11) A 12-year-old female complains of abnormality in her face that her eyes
are too far from each other and her nose is misshapen. She avoids people
because she doesnt want them to notice the abnormality. Doctors say that
she had nothing wrong. The patient has:
1 +Body dysmorphic disorder
2 Obsessive compulsive disorder
-- preoccupation with imagined defect in appearance or excess
concern around a slight anomaly
12) Difference between malingering & conversion is:
1 +Malingering refuses treatment & evaluation while conversion is
eager for treatment
2 Malingering is subconscious & conversion deliberately feigning sign
3 Malingering looks for primary gain, conversion looks for secondary
gain
4 Malingering are cooperative & friendly, conversion are non
cooperative
-- malingering is consciously done and not cooperative. Conversion
looks for both.

13) A 72-year-old female was brought in because she didnt eat anything since 2
days. She resists to be moved, she is motionless, and doesnt respond to stimulus
(verbal or non-verbal) and she previously said things that dont make sense. The
patient most likely has:
1.
2.
3.
4.
5.

Schizoaffective
+Catatonia
Cataplexy
Delusion disorder
Schizophreniform

Catatonic type : at least 2 of the following : (1)motor immobility (2)excessive


purposeless motor activity (3)extreme negativism/mutism (4)peculiar voluntary
movement or posturing (5)Echolalia/Echopraxia
Cataplexy is a sudden and transient episode of muscle weakness accompanied by full
conscious awareness, typically triggered by emotions
Schizoaffective : Schizophrenia + mood disorder.
Schizophreniform : last 1-6 months
Delusional disorder : Nonbizzare delusion at least 1 month, not impaired life function

14) A 52-year-old male had depressed mood, lost weight, and recently hears his
dead father voice blaming him for his faults and telling him that he is useless. The
patient believes that the inside of his body is rotting. This patient should be:
1. +Started on SSRI & antipsychotic
2. Started on SSRI alone
3. Patient had acute attack of schizophrenia
The patient had depression with psychosis (auditory hallucination + delusions). So,
we treat both.
15) To note vocal tics when observed:

1. Echolalia
2. +Coprolalia
3. Palilalia
Coprolalia : repetitive speaking of obscene words.
In the book, vocal tics include Coprolalia and Echolalia.??

16) A female complains that she sees little people walking around her house, she
also saw one of these little people in the doctor's clinic while waiting. This is
called:
1. +Hallucination
2. Hallucinogen
3. Illusion
Hallucination : Perception in absence of an external stimulus.
Illusion : misinterpretation of an external stimulus.

17) A female doesnt remember much about her childhood except that she was
taken away from her native parents because of abuse & neglect. Now she doesnt
account for a lot of hours & even days of her life. She finds cloths in her closet
she haven't bought, and her friends feel strange about her because she sometimes
acts childish and other times she is very aggressive. The patient has:
1. +Identity dissociative disorder
2. Disassociated Amnesia
3. Korsakoff dementia
Identity Dissociative Dis. : (1)has 2 or more distinct identities (2)inability to recall
personal information of one personality
Dissociated Amnesia : amnesia only, inability to recall important personal info, and
impaired daily function.
Korsakoff dementia : neurological disorder due tu lack of thiamine (vit B1) or viral
encephalitis

18) An 11-year-old adopted boy, his biological mother was alcoholic, was in a special
school because he had learning difficulties. On measuring his IQ, it was 60. We
classify him in which type of mental retardation?
1. +Mild Mental Retardation
2. Moderate Mental Retardation
3. Severe Mental Retardation
*from wiki, severity of intellectual disability. (different from the book)
Type
IQ

Mild
69-50

Moderate
49-35

Severe
34-20

Profound
<20

19) While examining a patient for MSE, speech was articulated & well formed but
not understood because words were from patient's own invention. This is called:
1. +Neologism
2. Word salad
Neologism : made-up word
Word salad : incoherent collection of words (real word)

20) A patient has taken an overdose of opioid. To overcome autonomic


dysregulation, you use:
1. +Clonidine
2. Respiridone
Clonidine : sympatholytic medication
Respiridone : atypical antipsychotics
21) Sustained posture is called:
1. +Catalepsy
2. Stereotyped
3. Cataplexy

Catalepsy is a nervous condition characterized by muscular rigidity and fixity of


posture regardless of external stimuli, as well as decreased sensitivity to pain.[1]
Cataplexy is a sudden and transient episode of muscle weakness accompanied by full
conscious awareness, typically triggered by emotions
A stereotypy is a repetitive or ritualistic movement, posture, or utterance.

22) Symptom present in mania but not in depressive episodes:


1. Overactivity
2. Irritability
3. +Flight of ideas
Depression got thought block, never flight of ideas.
23) Poverty of words is called:
+Alogia

24) Patient makes up words is called:


+Neologism
25) The most likely drug that causes dementia after prolonged use:
1. Phencyclidine
2. Opioid
3. +Inhaler *it causes permanent CNS damage
Not inhaler for asthma, inhalants .

26) The patient says: Im son of the King; this is called:


1. Hallucination
2. +Delusion
Delusion : false-belief

27) A young male with chronic schizophrenia, presented with fever, rigidity, and
leukocytosis. The first drug to start is:
1. Benztropine
2. Lorazepam
*This is neuroleptic malignant syndrome; the treatment is most importantly to
discontinue his medications and supportive care (hydration & cooling). Drugs that
can be used are sodium dantrolene, bromocriptine and amantadine, but are used
infrequently because of their own side effects and unclear efficacy. First Aid
"Benzodiazepines may be used to control agitation, but not the first drug!" -Wiki
28) One is a "thought content" disorder:
1. Tangentiality
2. Loss of association
3. +Obsession
Thought content : (1)Poverty of thought vs Overabundance (2)Delusions
(3)Suicidal/Homicidal thought (4)Phobias (5)Obsession (6)Compulsion.

29) A 72-year-old female developed 2nd to 3rd degree burns in 35% of her body
after her house was fired. Few days later she removed the IV line and started to
have psychotic symptoms. This is mostly due to:
1. +Delirium
2. Drug-induced
3. Acute stress disorder
Delirium : (1) disturbances of consciousness (2)change in cognition/development of
perceptual disturbances (3)disturbances develop over short period of time & tends
to fluctuate during the course of the day.
30) A drug that is used in OCD:
1. Mirtazapine
2. +Clomipramine
3. Buspirone
First line TT : SSRIs, but TCA(clomipramine also effective)

Mirtazapine: atypical antidepressant


Buspirone : Non-BDZs anxiolytics
31) A drug that causes thirst and metalic taste:
+Lithium

32) Alcohol abuse is most associated with:


1 +Major depression
2 Antisocial personality disorder
3 Panic disorder
33) Schizoid and schizotypal personality disorder share the following
symptom:
1 +Have no relationships
2 Magical thinking

*both are in the same cluster A


schizoid - neither desire nor enjoys close relationship, prefers to be alone (no
psychotic association)
shchizotypal - eccentric behaviours, peculiar thought patterns, magical thinking,
unusual perceptual experiences

34) According to DSM-4, trichotillomania is classified as:


1 +Impulse control disorder
2 Part of OCD
3 Personality disorder
*In DSM-5, it is now classified under Obsessive Compulsive Disorders.
* trichotillomania - an impulse disorder characterized by the compulsive urge to
pull out one's hair, leading to noticeable hair loss and balding, distress

35)A female delivered a baby, after few days she felt guilty as she cant
take over good maternal care to her child. This is called:
1 Postpartum depression - within 4w postpartum, last 2-6 month.
suicidal and infanticidal ideation. Tx: psychotherapy, maternal
SSRI.
2 +Postpartum blues - begins 2-4 d postpartum, usually last 48h.
mo medication needed, but increased risk of having PPD
3 Postpartum psychosis
36) Positive psychotic symptoms for 2 months:
+Schizophreniform disorder
brief psychotic - <1mo
schizophreniform - 1-6 mo
schizophrenia - >6mo

37)A personality disorder associated with psychotic symptoms:


1 Avoidant - socially awkward, fear of criticism
2 Dependent - excessive need to be taken care, fear of separation,
clinging
3 +Borderline - unstable moods, Hx of suicide
4 Narcissistic - sense of superiority, lack of empathy, needs
admiration
5 Histrionic - attention seeker, emotional, dramatic, often sexually
inappropriate
38) Negative psychotic symptoms for more than 7 months:
+Schizoaffective

*Actually the correct answer depends on the options.

39)A man who changed many jobs because he hadstolen money from his
workplace. In interview,he said that he feels guilty but cant change his
habit. The most likely diagnosis is:
1 Dissociative fugue - sudden, unexpected travel away from home
with inability to recall past identity, may assume new one
2 Antisocial personality - lack of remorse for action, manipulative,
deceitful, charming at first
3 +Pathological gambling
40)Every four or five weeks, a usually well-functioning and mild mannered
35-year-old woman experiences a few days of irritability, tearfulness, and
unexplained sadness. During these days, she also feels fatigued and bloated
and she eats large quantities of sweets. What is the most appropriate
diagnosis?
1 Cyclothymia - characterized by numerous mood swings, with periods of
hypomanic symptoms that do not meet criteria for a hypomanic episode,
[1]
alternating with periods of mild or moderate symptoms of depression that do
not meet criteria for a major depressive episode.

2 Borderline personality disorder


3 Dissociative identity - disorder dissociation so severe that the
usually integrated function of consciousness and perception
breaks down
4 +Premenstrual dysphoric disorder
5 Minor depressive disorder - mood disorder that does not meet full criteria
for major depressive disorder but in which at least two depressive symptoms are
present for two weeks.

41)Common symptom of depression in children is:


1 +Irritability
2 Sleepwalking

3 Enuresis
42)A 9 months old girl was developing normally, but in the last period her
parents noticed that she stopped developing and had decreased head
circumference. This is most likely:
+Rett's disease
Rett syndrome is a rare genetic disorder that affects the way the brain develops.
It occurs almost exclusively in girls. (Slowed growth, Loss of normal movement and
coordination, Loss of communication and thinking abilities, Abnormal hand
movements, Unusual eye movements, Breathing problems, Irritability )

43)Neuroleptic malignant syndrome, which drug to give?


1 +Bromocriptine(or dantrolene or amantadine)
2 Benztropine
3 Lorazepam
*Neuroleptic malignant syndrome (NMS) is a life-threatening neurological disorder
most often caused by an adverse reaction to neuroleptic or antipsychotic drugs.
(cardinal symptoms: Severe muscular rigidity, Hyperthermia (temperature >38C),
Autonomic instability, Changes in the level of consciousness)

44) Schizotypal personality is associated with:


+Family history of schizophrenia

45)A 70 years old hypertensive patient after hip surgery started speaking
abnormal words out of the blue and had weakness in his right hand:
+Multi-infarct dementia

46) When you are asking the patient "Are you doing things in a particular
way?", you mean for?
+Preoccupation - ideas which come againand again preventing
normal activity

47) Down syndrome patients have risk of developing:


+Alzheimer

48)A female takes steroid high dose for asthma and now has impaired
glucose test, abdominal obesity and striae (Cushing features). Most likely
her compliant is due to:
1 +Mood disorder due to substance abuse (medications)
2 MDD
3 GAD
49) Repeating words or phrases:
+Echolalia

50) According to DSM-4, which of the following is associated with both


schizoid and schizotypal personality disorder?
1 Odd believes and magical thinking
2 +No close relationship
*The correct answer depends on the other options.

*both cluster A - Mad

51) Type of schizophrenia with late onset and good function:


+Paranoid

52) Obsessions are:


+Thought content

* obsession - recurrent, persistent, intrusive thoughts, images or urge that


keep coming to mind despite the patient's effort to exclude them

53) A 70 years old male with a dementing disorder dies in a car accident.
During the previous five years, his personality had dramatically changed and
he caused much embarrassment to his family due to his intrusive and
inappropriate behavior. Pathological examination of his brain shows frontotemporal atrophy, gliosis of the frontal lobes white matter, characteristic
intracellular inclusions, and swollen neurons. Amyloid plaques and
neurofibrillary tangles are absent. Choose the correct diagnosis:
1
2
3
4
5

Alzheimers disease
+Picks disease
Creutzfeldt-Jakob disease
B12 deficiency dementia
HIV dementia

Points that suggest the answer are:


-frontotemporal dementia

-personality change occurs before any form of memory loss (Alzheimer


typically cause memory loss first)
-other symptoms are: difficulty in language and thinking, efforts to
dissociate from family, behavioral changes, unwarranted anxiety, impaired
regulation of social conduct
54) A young man smells burnt rubber, then he turns his head and
his upper body right, makes chewing movements, and fumbles with
his clothes. During the episode, which lasts one minute, he appears
dazed. Choose the most appropriate diagnosis:
1
2
3
4
5

Frontal lobe tumor


Derealization disorder
Conversion disorder
Absence seizure
+Partial complex seizures

Points that suggest the answer are:


-Complex partial seizures last 1 to 2 minutes
-An aura may occur at the beginning of a seizure. It may consist of a
strange smell, taste, sound, or visual disturbance, an unexplained feeling of
fear or anxiety, or a sense that everything seems strangely familiar, like it
has all happened before (dj vu), or strangely unfamiliar (jamais vu).
-Complex Partial Seizures include automatisms (such as lip smacking,
picking at clothes, fumbling), unaware of surroundings or may wander
Why the others are not the answer:
-Derealization disorder is alteration in
the perception or experience of the external world so that it seems unreal
- Conversion disorder is patients to suffer from neurological
symptoms, such as numbness, blindness, paralysis, or fits without a definable

organic cause. It is thought that symptoms arise in response to stressful


situations affecting a patient's mental health
- Absence seizure lasts between 10 and 30 seconds

55) A 45-year-old woman with a chronic mental illness seems to be


constantly chewing. Her tongue darts in and out of her mouth and
occasionally she smacks her lips. She also grimaces, frowns, and blinks
excessively. These abnormal movements are seen, characteristically, in:
1
2
3
4
5

Tourettes syndrome
Akathisia
+Tardive dyskinesia
Parkinsons disease
Wilsons disease

Points that suggest the answer are:


-Tardive dyskinesia is characterized by repetitive, involuntary,
purposeless movements like Grimacing, Tongue movements, Lip smacking,
Lip puckering, Pursing of the lips, Excessive eye blinking
Why the others are not the answer:
- Tourette syndrome is a nervous system disorder that starts in
childhood. It involves unusual repetitive movements or unwanted sounds that
can't be controlled (tics)
- Akathisia is movement disorder characterized by a feeling of
inner restlessness and a compelling need to be in constant motion, as well as
by actions such as rocking while standing or sitting, lifting the feet as if
marching on the spot, and crossing and uncrossing the legs while sitting.
People with akathisia are unable to sit or keep still, complain of restlessness,
fidget, rock from foot to foot, and pace.

- Parkinsons disease consist of tremor, rigidity, slowness of movement,


and postural instability
- Wilson disease initially have mild cognitive deterioration and
clumsiness, as well as changes in behavior. Then follow by the form
of parkinsonism
56) A 72 years old retired English professor with a long history of
hypertension has been having difficulties with tasks he used to find easy and
enjoyable, such as crossword puzzles and letter writing, because he cannot
remember the correct words and his handwriting has deteriorated. He has
also been having difficulties remembering the events of previous days and he
moves and thinks at a slower pace. Subsequently, he develops a mild right
facial hemiparesis and slurred speech. Your diagnosis is:
1
2
3
4
5

Wernicke Korsakoff syndrome


Huntingtons disease
Alzheimers disease
+Multi-infarct dementia
Creutzfeldt-Jakob disease

Points that suggest the answer are:


- Multi-infarct dementia is caused by a series of small strokes. Multiinfarct dementia is also called vascular dementia.
-Hypertension is one of the its risk factor. Others are diabetes,
atherosclerosis, smoking

57) As soon as she returns home from the hospital, three days after
delivering a healthy baby girl, a 23 year old woman becomes increasingly
irritable and tearful. She constantly worries about the baby, fearing she
wont be an adequate mother. What is the most likely diagnosis?
1 +Postpartum depression
2 Postpartum psychosis

3 Adjustment disorder
4 Postpartum blues
5 Major depressive episode

-Postpartum depression depressed affect, anxiety , poor concentration


starting within 4 weeks after delivery ,Last 2 weeks to a year or more
-Postpartum Psychosis is considered a medical emergency due to the
potential for a mom to harm herself or her baby
-Postpartum blues by depressed affect, tearfulness, fatigue starting
2-3 days after delivery, usually resolves within 10 days
(Jawapan yang diberi adalah 1. Tapi boleh jadi jugak 4 sebab dia cakap start
three days after delivery. Dan soalan x beritahu symptoms tu last sampai
berapa hari)

58) A 34-year-old secretary climbs 12 flights of stairs every day to reach


her office, because she is terrified by the thought of being trapped in the
elevator. Her diagnosis is:
1
2
3
4
5

Social phobia
Performance anxiety
Generalized anxiety disorder
+Specific phobia
Agoraphobia

Points that suggest the answer are:


- specific phobia kind of anxiety disorder that amounts to an
unreasonable or irrational fear related to exposure to specific objects or
situations ( in this case, specific to elevator)

Why the others are not the answer:


- Social anxiety disorder (SAD), also known as social phobia, is
an anxiety disorder characterised by an intense fear in one or more social
situations causing considerable distress and impaired ability to function in at
least some parts of daily life
- Stage fright or performance anxiety is the anxiety, fear, or
persistent phobia which may be aroused in an individual by the requirement
to perform in front of an audience
- Generalized anxiety disorder (GAD) is an anxiety disorder that is
characterized by excessive, uncontrollable and often irrational worry, that
is, apprehensive expectation about events or activities
- Agoraphobia is an anxiety disorder characterized by anxiety in
situations where the sufferer perceives certain environments as dangerous
or uncomfortable, often due to the environment's vast openness or
crowdedness

59) A 23 year old woman arrives at the emergency room complaining


that, out of the blue, she had been seized by an overwhelming
fear, associated with shortness of breath and a pounding heart. These
symptoms lasted for approximately 20 minutes and, while she was
experiencing them, she feared that she was dying or going crazy. The
patient has had four similar episodes during the past month
and she has been worrying that they will continue recurring. The most likely
diagnosis is:
1
2
3
4
5

Acute psychotic episode


Hypochondriasis
+Panic disorder
Generalized anxiety disorder
Post traumatic stress disorder

Points that suggest the answer are:


- Minimum 1 month
-spontaneous recurrent panic
- period between 20-30 minutes
- symptom attack (shortness of breath, a pounding heart, fear of dying
or going crazy )

60) After witnessing a violent argument between her parents, a young woman
develops sudden blindness, but she does not appear as distraught as would
be expected. Her pupils react normally to light and she does get hurt when
she trips over obstacles. Her parents, who are in the middle of a bitter
divorce, put aside their differences to focus on their daughters illness.
What is the most appropriate diagnosis?
1
2
3
4
5

Factitious disorder
Malingering
Somatization disorder
+Conversion disorder
Histrionic personality disorder

Points that suggest the answer are:


- A conversion disorder causes patients to suffer from neurological
symptoms, such as numbness, blindness, paralysis, or fits without a definable
organic cause. It is thought that symptoms arise in response to stressful
situations affecting a patient's mental health. Onset always preceded or
exacerbated by a stressor.

Why the others are not the answer:

- Factitious disorder is the patient intentionally produce medical or


psychological symptoms in order to assume the role of a sick patient.
- Malingeribg disorder is the patient feigning of physical or
psychological symptoms in order to achieve personal gain.
- Somatization disorder is the patients presented with multiple vague
symptoms, Long standing history of numerous visits to clinics,
Symptoms cant be explained by any medical disorder.
- Histrionic personality disorder is a pervasive pattern of excessive
emotionality and attention seeking, They are dramatic, unable to form
long lasting meaningful relationships.

61) Which is not a perceptual disturbance?


1 Derealization
2 Depersonalization
3 Illusion
4 +Delusion
5 Hallucination

62) A 28-year-old taxi driver is chronically consumed by fears of having


accidentally run over a pedestrian. Although he tries to convince himself
that his worries are silly, his anxiety continues to mount until he drives back
to the scene of the accident and proves to himself that nobody lies hurt in
the street. This is an example of:
1
2
3
4

+A compulsion secondary to an obsession


An obsession triggered by a compulsion
A delusional ideation
A typical manifestation of obsessive-compulsive personality
disorder

5 Phobia

Points that suggest the answer are:


- Obsession ( repetitive and intrusive thought, feeling, or idea that is
egodystonic) comes first before the compulsion (repetitive behavior that
linked to the obsession when performed can relieve the anxiety caused by
the obsession)
63) A consult is requested for a 75-year-old female who underwent hip
repair two days earlier, because the nurses noted that from time to time the
patient was not making any sense. The psychiatric resident conducting the
evaluation observes that the patient believes she is in 1954, & still living at
her son home. She believes the resident is her deceased father. These
statements are manifestations of impaired:
1
2
3
4
5

Concentration
Memory
Thought associations
+Orientation
Level of consciousness

Points that suggest the answer are:


- Orientation to person, place and time: Who are you? Where you
right now? What time is it? What day is today?

Why the others are not the answer:


- Concentration: If you have 100 JD and you buy something worth 7
JD, how much o you have left? And then subtract it again with 7JD?
Can you mention the days of the week reversibly? (Monday-SundaySaturday)

-Memory: Immediate : Say something (eg your name) and ask him 5
minutes later, Recent: What did you have for breakfast today? For
your dinner last night?, Remote : Do you remember your childhood?
War? Known past events?
- Thought association: Thought insertion / withdrawal / broadcasting
64) Fluoxetine is a:
1 MOA inhibitors
2 TCA antidepressant
3 +SSRI antidepressant

SSRI: Paroxetine, Fluoxetine, Cotalopram, Escitalopram


MOA inhibitor: isocarboxazid, tranylapromine
TCA antidepressant: phenelzine, imipramine, amytryptyline

65) Which of the following is a finding in imaging of schizophrenia patient?


Frontal lobe enlargement
2 Frontal lobe decreasing size
3 +Ventricles enlargement
1

What Causes Schizophrenia? The exact cause is not known, but scientists suspect genes and
environment both play a role. Inside the brain, levels of the chemical messengers dopamine and
glutamate may be out of balance. And brain structures may be abnormal, too. For example, brain
scans of identical twins show that the

fluid-filled "ventricles" can be larger in a twin with

schizophrenia, compared with a twin who does not have the illness. Activity levels can also be higher
or lower than normal in some areas of the schizophrenic brain.

66) Delusion is a:
+Thought content disorder
2 Perception disorder
1

Definition : Delusions are false or erroneous beliefs that usually involve a misinterpretation of perceptions
or experiences. Their content may include a variety of themes (e.g., persecutory, referential, somatic,
religious, or grandiose).

67) A speech with lacking goal directed with lacking details is known as:
1 Circumstantiality

-Citing excessively minute details which distract from the central theme of a conversation. Example:
When asked about a bruise on her arm, the patient recounts everything else that happened that same
day before explaining how she was injured.

2 +Tangentiality
-Replying to questions with digressive or otherwise irrelevant speech. Example: The patient begins to
explain how she was injured, but loses her train of thought and goes on to other subjects.

68) Which is a subcortical dementia?


Vascular dementia
2 +Dementia due to Parkinsons (also Huntingtons Disease)
1

3 Dementia due to CJD


4 Dementia due to Alzheimers (cortical / cerebral cortex)
5 Dementia due to Picks Disease
69) Delusion that is not explained by psychological processes is called
primary delusion. What is the type of delusion?
1
2
3
4

Nihilistic
Poverty
Guilt
+Thought insertion

Primary delusions (sometimes called true delusions) are distinguished by a transformation of


meaning, so that the world, or aspects of it, are interpreted in a radically different way by the delusional
person. To others, this intepretation is 'un-understandable' in terms of the normal mental causality, mood,
environmental influences and other psychological or psychopathological factors.

4 types of primary delusion:


1.Delusional intuition- where delusions arrive 'out of the blue', without external cause.
2.Delusional perception - where a normal percept is interpreted with delusional meaning. For
example, a person sees a red car and knows that this means their food is being poisoned by the police.

3. Delusional atmosphere - where the world seems subtly altered, uncanny, portentous or sinister.
This resolves into a delusion, usually in a revelatory fashion, which seems to explain the unusual feeling of
anticipation.

4. Delusional memory - where a delusional belief is based upon the recall of memory or false memory
for a past experience. For example, a man recalls seeing a woman laughing at the bus stop several weeks
ago and now realises that this person was laughing because the man has animals living inside him.

Secondary delusions (sometimes called delusion-like ideas) are considered to be, at least in principle,
understandable in the context of a person's life history, personality, mood state or presence of other
psychopathology. For example, a person becomes depressed, suffers very low mood and self-esteem,
and subsequently believes they are responsible for some terrible crime which they did not commit.

70) Agoraphobia can be associated with major depression and anxiety, and
can be associated with all of the following, except:

+Demoralization (Demoralization, as described by Jerome Frank, is experienced as a persistent


inability to cope, together with associated feelings of helplessness, hopelessness, meaninglessness, subjective
incompetence and diminished self-esteem.)

*Question actually cannot be answered except when having all the other options.

71) A female patient came to the clinic was taking a medication (she couldnt
remember). She complained of increasing urination and thirst. What is the
most likely diagnosis?
1 Major depression
2 +Bipolar disorder (side effects of Lithium)

3 Panic attack
Common side effects of lithium can include:
Hand tremor (If tremors are particularly bothersome, an additional medication can help.)
Increased thirst Increased urination
Diarrhea
Vomiting
Weight gain
Impaired memory
Poor concentration
Drowsiness
Muscle weakness
Hair loss
Acne
Decreased thyroid function (which can be treated with thyroid hormone

72) Which is not an alcoholic withdrawal symptom?


1
2
3
4

Nausea & vomiting


Transient hallucination
+Hypersomnia
Autonomic dysregulation

Minor alcohol withdrawal symptoms often appear 6 to 12 hours after a person stops drinking. Sometimes a
person will still have a measurable blood alcohol level when symptoms start. These symptoms include: Shaky hands
Sweating Mild anxiety Nausea Vomiting Headache Insomnia
Between 12 and 24 hours after they stop drinking, some patients may experience visual, auditory, or tactile
hallucinations. These usually end within 48 hours. Although this condition is called alcoholic hallucinosis, it's not the
same as the hallucinations associated with DTs. Most patients are aware that the unusual sensations aren't real.
Withdrawal seizures usually first strike between 24 and 48 hours after someone stops drinking, although they can
appear as early as 2 hours after drinking stops.. The risk of seizures is especially high in patients who previously have
undergone multiple detoxifications. DTs usually begin between 48 and 72 hours after drinking has stopped, Risk
factors for DTs include a history of withdrawal seizures or DTs, acute medical illness, abnormal liver function, and
older age.
Symptoms of DTs, which usually peak at 5 days, include: Disorientation, confusion, and severe anxiety Hallucinations
(primarily visual) which cannot be distinguished from reality Profuse sweating Seizures High blood pressure Racing
and irregular heartbeat Severe tremors Low-grade fever

73) A patient was having bulimia nervosa and depression. Shes complaining
of insomnia and apathy. Which drug should be avoided for this patient?
1
2
3
4

Amitriptyline
Imipramine
Trazodone
+Bupropion (because of higher reported incidence of seizures

in such patients treated with the drug)


5 Fluoxetine

74) Which is a positive prognosis for a patient with schizophrenia?


1
2
3
4

+Affective symptoms
Male
Many relapses
Gradual onset

GOOD PROGNOSIS : A diagnosis of paranoid schizophrenia versus one of the other subtypes; this subtype
tends to respond better to treatment and paranoid schizophrenics usually have an overall higher level of functioning
than other subtypes; Being a female women typically have a later onset and also tend to respond better to
neuroleptics; Having fewer negative symptoms; No family history of schizophrenia; High level of functioning prior to
onset; Acute onset; Older age of onset; A good support system; Shorter period of active symptoms;

POOR PROGNOSIS : Earlier age of onset; Being a male; A higher number of negative symptoms; A family
history of schizophrenia; A low level of functioning prior to onset; Poor or no support system; A history of substance
abuse

75) Which is not a cause of anxiety?


1
2
3
4

+Hypothyroidism
Pheochromocytoma
Hyperthyroidism
Hypoparathyroidism

76) In evidence based medicine, schizophrenia patient shows improvement in


functional ability due to which type of treatment:
1
2
3
4

Supportive
Psychodynamic
Psychoanalysis
+Cognitive behavior (strongest evidence based!)

*Wiki says:
Several meta-analyses have shown CBT to be effective in schizophrenia, and
the American Psychiatric Association includes CBT in its schizophrenia guideline as
an evidence-based treatment.

And Medscape says:


Cognitive therapy may be effective as a stand alone treatment for schizophrenia.
For schizophrenic patients who cannot or will not take antipsychotic medication,
cognitive therapy may be the most viable option.

According to data from the first randomized trial of cognitive therapy as a stand
alone therapy for schizophrenia, structured treatment with a therapist
significantly reduced the severity of psychiatric symptoms and improved personal
and social functioning and some dimensions of delusional beliefs and voice hearing.

77) Seen in early imaging of Alzheimer dementia:


1 + Hypocampal atrophy
2 Diffuse cortical atrophy
3 Sulcal widening
Alzheimer disease is characterised by the accumulation within the brain of senile (neuritic) plaques, neuritic
(neurofibrillary) tangles, and progressive loss of neurons 2. The progression of pathology initially involves the
transentorhinal region and then spreads to the hippocampal complex and mesial temporal lobe structures and
eventually the temporal lobes and basal forebrain 1. The underlying reason for accumulation of senile (neuritic)
plaques and neurofibrillary tangles remains poorly understood, as does the reason for non uniform distribution in the
cortex.

78) A female patient complained of dizziness when waking in the morning and
standing. She is on amitriptyline for depression. The cause of her symptom is
due to:
1 Dehydration
2 Hypoglycemia
3 +Alpha adrenergic blockade (?)
Note : dizziness is usually caused by a drop in blood pressure when
suddenly changing position. Patients should slowly rise from a sitting or
lying position if dizziness is noticed.

79) Which of the following presents with more cognitive preoccupations


rather than physical symptoms?
1 Pain disorders
2 +Hypochondriasis When you have hypochondria, you become obsessed with the idea that
you have a serious or life-threatening disease that hasn't been diagnosed yet. This causes significant

anxiety that goes on for months or longer, even though there's no clear medical evidence that you
have a serious health problem

3 Conversion disorders
4 Somatization disorders
5 Undifferentiated somatoform disorders
Somatization Disorder Patients with somatization disorder (formally called hysteria or Briquet's syndrome)
typically have a long history of going to the doctor for many different unexplainable symptoms.
This pattern of symptoms has occurred for many years and began before they were 30 years old. Their symptom
history must include various pain issues, gastrointestinal problems (e.g. diarrhea or vomiting), sexual symptoms (e.g.
low libido), and symptoms that would suggest a neurological problem (e.g. paralysis or seizures).

1. Undifferentiated Somatoform Disorder This is similar to somatization disorder, except that the patient must
complain of at least one unexplainable symptom for at least 6 months. Common complaints include digestive
problems and chronic fatigue.

2. Conversion Disorder Individuals with this disorder have symptoms or difficulties with their senses (e.g.
blindness, deafness) or their motor functioning (e.g. difficulties swallowing, weakness in a specific area). Their
symptoms are pseudoneurological, which means they suggest a neurological cause but no such cause can be
found. Prior to the onset or worsening of their symptoms they experienced conflict or other types of stress that is
believed to be associated with the development of the disorder.

3. Pain Disorder As the name suggests, pain is the primary complaint with this disorder. There is no physical
explanation for the pain. Underlying psychological issues are believed to play a role in triggering, maintaining, or
exacerbating the pain or making it more intense.

4. Hypochondriasis Individuals with this disorder (often called hypochondriacs by those who know them) are
preoccupied with the belief or fear that they have a serious medical condition. Their belief or fear is triggered by their
own misinterpretation of their physical symptoms or bodily functions (e.g. they have occasional headaches and fear
they have a brain tumor).

5. Body Dysmorphic Disorder Individuals with this disorder become extremely preoccupied with and distressed
about one or more imagined or actual (but minor) physical flaws (e.g. someone with a small scar on her hand always
wears gloves or keeps her hand hidden from others).

6. Somatoform Disorder not Otherwise Specified (NOS ) This is essentially a catch-all diagnosis for anyone
who meets the basic criteria for a somatoform disorder but doesnt meet the criteria for one of the other somatoform
disorders

80) The neurotransmitters affected in delirium:

1 +Acetylcholine and dopamine (?)


2 Norepinephrine and serotonin
3 Norepinephrine and dopamine
Reduced cholinergic function, excess release of dopamine, norepinephrine, and glutamate, and both
decreased and increased serotonergic and gamma-aminobutyric acid activity may underlie the different
symptoms and clinical presentations of delirium.

81) A patient has factitious disorder have association with all except:
+Malingering *Question actually cannot be answered except when

having all the other options

Factitious disorder is a serious mental disorder in which someone deceives others by appearing sick, by
purposely getting sick, or by self-injury. Factitious disorder symptoms can range from mild (slight exaggeration of
symptoms) to severe (previously called Munchausen syndrome). The person may make up symptoms or even
tamper with medical tests to convince others that treatment, such as high-risk surgery, is needed

82) Middle-aged female patient with a full remission of alcohol, came in a


history of generalized anxiety disorder and her primary physician prescribed
for her a short acting benzodiazepine. She came to you -her psychiatristwhat do you do?
Add SSRI
2 +SSRI monotherapy (SSRI is the treatment for GAD along
with cognitive behavioral therapy)
3 Buspiron monotherapy
4 Replace with long acting benzodiazepine
5 Keep her on the short acting benzodiazepine
*benzodiazepine and alcohol can cause the GAD
1

83) A patient with low self confidence, preoccupied by being alone, can't
take decisions: which cluster?
1 A

2 B
3 +C (criteria for dependent personality disorder)

84) After delivering a full healthy child the mother started to be agitated
and hearing voices telling her to kill her baby. Which of the following is
right?
Start her on antipsychotics and antidepressants
2 +Take the baby immediately away from her (to ensure the
safety and healthy development of both the baby and its
mother)
3 It's a picture of schizophrenia
1

85) A patient afraid of having heart attack, he had sweating, palpitations, all
his investigations where normal, what's the diagnosis?
+Panic attack without agoraphobia

86) Why dissociative disorders happen?


+Separation of implicit and explicit memory
* Dissociative disorders (DD) are conditions that involve disruptions or
breakdowns of memory, awareness, identity, or perception. People with
dissociative disorders use dissociation, a defense mechanism, pathologically
and involuntarily. Dissociative disorders are thought to primarily be caused
by psychological trauma
87) Conversion syndrome may come in all, except:
1 Blindness
2 Paralysis
3 +Pain
* A conversion disorder causes patients to suffer from neurological symptoms,
such as numbness, blindness, paralysis, or fits without a definable organic

cause. It is thought that symptoms arise in response to stressful situations


affecting a patient's mental health.

88) A 10 years old boy with mania, what is the drug?


1 Divalproex
2 Lithium
*As the choices and the question stem are not complete, we can't answer!
Medscape:
Risperidone is approved for bipolar mania in children aged 10-17 years.
Quetiapine is approved for bipolar mania in children aged 10-17 years.
Lithium and divalproex are approved for 12 years and older.

89) Something not seen in OCD:


+Body-dysmorphic disorder ??
*In DSM-V, body-dysmorphic disorder is classified under OCD.

90) A patient had depression with psychotic features. After treatment of 3


months, all symptoms disappeared. What to do?
1 Continue drugs for 6-9 months
2 +Continue antidepressant and taper antipsychotic
3 Continue antidepressant and stop antipsychotic
4 Stop both
5 Continue antipsychotic and taper antidepressant

91) A pregnant lady is taking valproic acid. What is the deformity you expect
to see by ultrasound?
1 +Spina bifida
2 Renal hypoplasia
3 Tetralogy of Fallot
4 Microcephaly
*6 birth defects linked to valproic acid (spina bifida, Atrial septal defect,
cleft palate, Hypospadias, Polydactyly and Craniosynostosis)
92) Difference between panic disorder and GAD:
+GAD is more chronic and less fluctuation

93) Difference between unipolar and bipolar:


1
2
3
4

More suicide
+Later onset Not Sure
More gradual
More substance abuse

*The question is not really clear!

94) A patient with undemented depression. What investigations you should


do?
+B12 and folic acid

95) One is true about malingering:

+Presentation in medico-legal aspect is common


* malingering is fabricating or exaggerating the symptoms
of mental or physical disorders for a variety of "secondary gain" motives,
which may include financial compensation (often tied to fraud); avoiding
school, work or military service; obtaining drugs; getting lighter criminal
sentences; or simply to attract attention or sympathy.

96) A phsycotic patient present to ER having fever, and leukocytosis, what is


the treatment:
+ Bromocriptine
* Neuroleptic malignant syndrome (NMS)

97) Long use of SSRIs in female patients can cause:


Insomnia
2 +Anorgasmia
3 Headache
1

98) Not a special phobia:


+Speak in public (social phobia)

99) Suggestive of post traumatic attack:


+Detachment

*Question actually cannot be answered except when having all the other options.

100) Lamotrigine is being evaluated for its efficacy in treating manic,


depressive and mixed bipolar disorders. Which of the following is correct?
1 Lacks an effect on manic episode

2 Effective in treating both mania and psychosis in bipolar


3 Lacks an effect on rapidly cycling bipolar
4 +Effective in treating depressive episodes of bipolar

101) Lonely, shy female, wishes she had more friends & a more active social
life but she fears that others will reject her. Her personality disorder is
described in:
1 Cluster A
2 Cluster B
3 +Cluster C (avoidant personality disorder)
4 Cluster D
5 Cluster E
102) Which one is a prognostic factor for a good outcome in schizophrenia?
1.
2.
3.
4.

+Accompanied by affective symptoms (negative symptoms)


Young age of onset
Living in an industrial nation
Male
Good prognosis
-Female
-late onset
-positive symptoms
-good social support

103) EBM (evidence-based medicine) shows that the best psychotherapy for
schizophrenia is:
1. Psychoanalysis
2. Supportive
3. Cognitive-behavioral (improve patients ability to function in
society)
4. Family therapy or group therapy or something like that

104) A patient with GAD (or was it panic disorder?) commonly experiences all
of the following, except:
1.
2.
3.
4.

+Demoralization
Addiction to stimulants
Addiction to alcohol
Depression

105) Least likely to be associated with a factitious disorder:


1.
2.
3.
4.

Depression
Schizophrenia
+Malingering
Substance abuse

Factitious disorder
-intentionally produce medical and psychological symptoms in order to
assume the role of sick patient.
Malingering
-feigning of physical or psychological symptoms in order to achieve personal
gain
106) All might be signs of conversion, except:

1.
2.
3.
4.

Loss of voluntary movement


Blindness
Aphonia : inability to produce voice
+Pain

Conversion: under topic Somatoform disorder


-have at least one neurological symptom that cannot be explained by medical
disorder.
107) In comparison with panic disorder, general anxiety is:
+More chronic with remission period
*The question is not complete!
Panic attacks can occur in GAD, arising out of escalating and uncontrollable worry:
but the presence of unexpected (uncued) panic attacks is unusual in GAD.
Patients with panic disorder tend to have episodic and calamitous thoughts about
presumed life-threatening acute illnesses, whereas patients with GAD focus more
persistently on less specific but more chronic complaints involving multiple organ
systems.- UpToDate

108) All are OCD types except:


1.
2.
3.
4.

+Body image distortion


Hoarding
Dirt & contamination
Counting & checking

*In DSM-V, body-dysmorphic disorder is classified under OCD

109) Dependant personality disorder is in:

1. Cluster A (schizoid,schizotypal, paranoid)


2. Cluster B (antisocial, borderline, histrionic, narcissistic)
3. +Cluster C (avoidant, dependent, OCPD)
110) Typical case of severe agitated patient in ER, which drug to give?
+Anti psychotic

111) Typical case of mild agitated patient in ER, which drug to give?
+Benzodiazepine

*The last two questions, the answers depend on the other choices which are missed
here!
Anyway, UpToDate says:
For severely violent patients requiring immediate sedation, give a rapidly acting
first generation (typical) antipsychotic (eg, droperidol) or benzodiazepine alone
(eg, midazolam) or a combination of a first generation antipsychotic and a
benzodiazepine (eg, droperidol and midazolam, or haloperidol andlorazepam)
For patients with agitation from drug intoxication or withdrawal, give a
benzodiazepine.
For patients with undifferentiated agitation, we prefer benzodiazepines, but
first generation antipsychotics are a reasonable choice.
For agitated patients with a known psychotic or psychiatric disorder, we prefer
first generation antipsychotic agents, but second generation antipsychotics are a
reasonable choice.

112) What are the neurotransmitters involved in delirium?

1. NE and Ach
2. NE and serotonin
3. +Ach and Dopamine (ach, dopamine, serotonin, GABA)

113) Most specific for schizophrenia:


1. Auditory hallucination
2. +Belief power as alien species

114) Why dissociative disorders happen:


Separation of implicit and explicit memory
(** Medscape says: Pathology of dissociative disorders
From a psychological perspective, dissociation is a protective activation of altered states
of consciousness in reaction to overwhelming psychological trauma. After the patient
returns to baseline, access to the dissociative information is diminished. Psychiatrists have
theorized that the memories are encoded in the mind but are not conscious, ie, they have
been repressed.
In normal memory function, memory traces are laid down in 2 forms, explicit and implicit.
Explicit memories are available for immediate and conscious recall and include recollection
of facts and experiences of which one is conscious, whereas implicit memories are
independent of conscious memory. Further, explicit memory is not well developed in
children, raising the possibility that more memories become implicit at this age.
Alterations at this level of brain function in response to trauma may mediate changes in
memory encoding for those events and time periods. Dissociation is also a neurologic
phenomenon that can occur from various drugs and chemicals that may cause acute,
subchronic, and chronic dissociative episodes.)

115) A case about a patient presented with acute alcohol intoxication, what
is the first line of management:
1 +Glucose

2 Thiamine
3 Diazepam
*UpToDate:)
As a general rule, all intoxicated patients should receive a rapid bedside glucose
determination, followed by dextrose infusion if hypoglycemia is present.
In addition, all patients presenting with acute ethanol intoxication should be
carefully assessed for occult traumatic injuries and should be asked whether they
have ingested or otherwise taken other drugs or potentially harmful substances.
Patients presenting with coma secondary to ethanol intoxication should receive at
least 100 mg of parenteral thiamine to prevent or treat Wernicke's
encephalopathy, along with dextrose.)

HAYAT (2013) Psychiatry Final Questions


Note: The exam was 45 questions, 85 minutes, one form
1. What is the treatment for benzodiazepine overdose?
+A. Flumazenil
(Flumazenil reverses the effects of benzodiazepines by competitive
inhibition at the benzodiazepine binding site on the GABA receptor)
2. Which of the following is a benzodiazepine?
+A. Clonazepam
B. Respirdione
(Eg. of benzodiazepine:Alprazolam,clonazepam,diazepam,lorazepam etc.)
3. Which of the following is characteristic of bulimia nervosa?
A. Amenorrhea
B. Psychotic symptoms
+C. Binge eating
D. Low body weight
Bulimia Nervosa: An eating disorder which is characterized by )
recurrent binge eating, followed by compensatory behaviors. Patients
are usually ashamed of their eating behaviors, tend to keep them
(.secret, and often maintain normal body weight
A patient who had myocardial infarction is diagnosed to have .4
?major depression. What drug would you prescribe
A. Trazodone
B. Bupropion
C. TCA
D. SSRI+
E. MAOI
SSRIs will reduce risk of MI. Only longer term use of SSRIs was)
associated with reduced MI risk, suggesting that other mechanisms,
(.besides an acute anti-platelet effect, may reduce MI risk

Which of the following antidepressants has the least sexual side .5


?effects
A. Paroxetine
B. Mirtazapine+
C. Fluvoxamine
D. Imipramine
Antidepressants with the lowest rate of sexual side )
(effects:Mirtazapine,Bupropion,Vilazodone
Which of the following symptoms is least associated with .6
?generalized anxiety disorder
A. Irritability
B. Stress incontinence+
:Generalized anxiety disorder)
Excessive anxiety and worry for at least 6 months-1
No realistic reasons to worry-2
Fear out of proportion to circumstances -3
Fear causing disruption to everyday functioning -4
Difficult to control worry-5
Associated with:(a) Restlessness (b) Autonomic arousal -6
Difficulty in concentration (d) Muscle tension (c)
(Irritability (f) Disturbed sleep (g) Easy fatigability (e)

7. Trauma to the dorsolateral frontal lobe is likely to lead to which of


the following?
A. Disinhibition
+B. Apathy
C. Apraxia
D. Affect lability
E. Euphoria
(Dorsolateral frontal lobe is concerned with planning,strategy formation, and
other executive functions.)

8. Which of the following is associated with poor prognosis in


schizophrenia?
A. Female patient
B. Acute onset
+C. Insidious onset
D. Presence of positive symptoms
(Poor prognosis in schizophrenia:
1- Early onset
2-Poor social support
3-Negative symptoms
4-Family history +ve
5-Gradual onset
6-Male sex
7-Many relapses
8-Poor premorbid functioning (social isolation, etc.)
9. Which of the following is true about atypical antipsychotics (SGA)?
+A. They block 5-HT2 receptors
B. Hyperprolactinemia is a characteristic side effect
C. Higher incidence of extrapyramidal side effects
10. What type of psychotherapy is best used in treating major
depression?
A. Supportive
B. Psychodynamic
+C. Interpersonal
D. Psychoanalysis
(Psychotherapy in MDD: behavioral, cognitive supportive, dynamic and family
therapy.)
11. Which of the following best describes the events leading to posttraumatic stress disorder (PTSD)?
A. Reaction only to death or physical injury
B. +Reaction to events outside the realm of normal human experience
C. Hopelessness

D. Terror and helplessness


(PTSD:
1-The symptoms last longer than one month
2-Features include:
(a)Intense fear, helplessness, horror
(b)Recurrent and intrusive distressing recollections of the
including images, thoughts, or perceptions.)

event,

12. In a person who wants to commit suicide, which of the following


indicates the highest risk?
A. Loneliness
B. Feeling of hopelessness
+C. Previous serious attempt

13. Which of the following is the main difference between anorexia


nervosa and bulimia nervosa?
A. Amenorrhea
+B. Lower body weight
C. Higher body weight
D. Dental erosions
E. Hand calluses
(Anorexia nervosa:low body weight, Bulimia nervosa:normal body weight.)
14. Odd, magical thinking and lack of interpersonal relations are
characteristic for:
+A. Schizotypal personality disorder
B. Schizoid
C. Schizophrenia
(Schizotypal personality disorder:
A pervasive pattern of eccentric behavior and thinking.-.1
2-They prefer social isolation and are develop severe discomfort in social
situation.
3-They have unconventional beliefs.)

15. Case about a female, mentions that she likes to look at things as all
good or all bad:
+A. Borderline personality
(Borderline personality disorder:
1-Patients have unstable moods, behaviors and interpersonal relationships.
2-They fear abandonment and have poor identity.
3-They are impulsive and have a history of suicide attempts.
4-They have a higher rate of physical, emotional, sexual abuse.)
16. A patient known to have dysthymia developed a major depressive
episode, what do we call this condition?
A. Bipolar I
+B. Double depression
C. Major depression
17. What drug is used to relieve symptoms of opioid withdrawal?
+A. Clonidine
(The signs and symptoms of opioid withdrawal include: lacrimation,
rhinorrhea, pupillary dilation, piloerection, diaphoresis, diarrhea,, mild
hypertension, tachycardia, fever, and insomnia ,A flulike syndrome
subsequently develops, with complaints, demands, and drug seeking.)
(Treatment of opioid withdrawal include: Naloxone,Clonidine)
18. Which of the following is NOT a manifestation of Wernickes encephalopathy?
A. Nystagmus
B. Gaze palsy
C. Confusion
D. Ataxia
+E. Confabulation

19. A patient with refractory major depression was scheduled for ECT. Which of the
following is a relative contraindication to ECT?
A. Hypertension

+B. Recent MI
C. Pregnancy
D. Degenerative joint disease
20. Which of the following is a common side effect of methylphenidate?
A. Tremor
+B. Insomnia
C. Weight gain s/e: decrease appetite, insomnia, weight loss
21. A 50 year old male who has problems with his sleep. Doesnt feel refreshed after
sleep, sleeps in inappropriate places. His wife reports he snores while sleeping. He also
complains of a dry mouth. Whats the most likely diagnosis?
+A. Obstructive sleep apnea
22. Which of the following is not a paraphilia?
A. Exhibitionism
B. Sadomasochism
+C. Gender identity disorder
23. A 5-year-old boy shows no interest in other children and ignoresadults
other than his parents. He spends hours lining up his toy cars orspinning their
wheels. He rarely uses speech to communicate. Physical examination indicates
that his head is of normal circumference and his gait is normal. Which of the
following is the most likely diagnosis for this boy?
A. Obsessive-compulsive disorder
B. Asperger syndrome
C. Childhood disintegrative disorder

+D. Autism
E. Rett disorder

24. A 65-year-old man, who had been hospitalized for an acute


pneumonia 3 days previously, begins screaming for his nurse, stating that
there arepeople in the room out to get me. He then gets out of bed
and beginspulling out his IV line. On examination, he alternates between
agitationand somnolence. He is not oriented to time or place. His vital
signs are as follows: pulse, 126 beats per minute; respiration, 32
breaths per minute; blood pressure (BP), 80/58; temperature, 39.2C
(102.5F). Which of the following diagnoses best fits this patients
clinical picture?
A. Dementia
B. Schizophreniform disorder
C. Fugue state
+D. Delirium
E. Brief psychotic episode

25. A 24-year-old man with chronic schizophrenia is brought to the


emergency room after his parents found him in his bed and were unable
tocommunicate with him. On examination, the man is confused and
disoriented.He has severe muscle rigidity and a temperature of 39.4C
(103F).His blood pressure is elevated, and he has a leucocytosis.
Which of the following is the best first step in the pharmacologic
treatment of this man?
A. Haloperidol
B. Lorazepam

+C. Bromocriptine
D. Benztropine
E. Lithium
*dx: neuroleptic malignant syndrome, rx : dantrolene, bromocriptine

26. A 54-year-old man with a chronic mental illness seems to be


constantly chewing. He does not wear dentures. His tongue darts in and
out ofhis mouth, and he occasionally smacks his lips. He also grimaces,
frowns, and blinks excessively. Which of the following disorders is most
likely in this patient?
A. Tourette syndrome
B. Akathisia
+C. Tardive dyskinesia
D. Parkinson disease
E. Huntington disease

27. A 25-year-old woman delivers a healthy baby boy by Caesarean


section. She notes over the next week that she has become irritable
and is notsleeping very well. She worries that her child will die and
fantasizes that if the child died, she would kill herself as well. She

reports not being able tosleep, and has lost 10 lb within 1 week. Which
of the following is themost likely diagnosis?
+A. Postpartum depression
B. Postpartum psychosis
C. Uncomplicated bereavement
D. Postpartum blues
E. Generalized anxiety disorder

28. A 19-year-old man is brought to the physician by his parents after


he called them from college, terrified that the Mafia was after him. He
reports that he has eaten nothing for the past 6 weeks other than
canned beans because they are into everythingI cant be too careful.
He is convinced that the Mafia has put cameras in his dormitory room
and that they are watching his every move. He occasionally hears the
voices of two men talking about him when no one is around. His
roommate states that for the past2 months the patient has been
increasingly withdrawn and suspicious.Which of the following is the most
likely diagnosis?
A. Delusional disorder

B. Schizoaffective disorder
+C. Schizophreniform disorder
D. Schizophrenia
E. Phencyclidine (PCP) intoxication

29. A 27-year-old woman has been feeling blue for the past 2 weeks.
She has little energy and has trouble concentrating. She states that 6
weeks agoshe had been feeling very good, with lots of energy and no
need for sleep. She says that this pattern has been occurring for at
least the past 3 years,though the episodes have never been so severe
that she couldnt work.Which of the following is the most likely
diagnosis?
A. Borderline personality disorder
B. Seasonal affective disorder
+C. Cyclothymic disorder
D. Major depression, recurrent
E. Bipolar disorder, depressed

30. A patient who is convinced that people in the bus are talking about
him. He also believes that the TV news presenter is communicating with
him, this is called:
+A. Delusions of reference
B. Grandiosity

31. A patient who gives excessive unrelated details to questions before


finally reaching his point, this is called:
+A. Circumstantiality

32. A 23-year-old woman arrives at the emergency room complaining


that, out of the blue, she had been seized by an overwhelming fear,
associated with shortness of breath and a pounding heart. These
symptoms lasted for approximately 20 minutes, and while she was
experiencingthem, she feared that she was dying or going crazy. The
patient has hadfour similar episodes during the past month, and she has
been worrying that they will continue to recur. Which of the following is
the most likelydiagnosis?
A. Acute psychotic episode
B. Hypochondriasis
+C. Panic disorder
D. Generalized anxiety disorder
E. Posttraumatic stress disorder

33. A young female refuses a job because it involves speaking in public.


She feels anxious that she might be humiliated or embarrassed in public
situations. Whats the most likely diagnosis?
+A. Social anxiety disorder
B. Agoraphobia

34. A young healthy adult is asked to give a urine sample for the annual
check-up. He cuts his finger and drops blood into the sample. Whats
the diagnosis?
A. Somatization disorder
+B. Factitious disorder
C. Malingering
D. Conversion disorder

*unlike malingering which has personal or secondary gain.

35. A 23-year-old woman presents to her physician with the chief


complaint that she is anxious about the way she looks. She notes that
for as long as she can remember, she has been obsessed about the
fact that something must be wrong with her face. She notes that her
eyes are too far apart and her nose is misshapen. She states that this
concern is ruining her life because she spends all her time isolated
from others so that they cannot see her face. The physician did not
notice anything unusual about the patientsface, but the patient cannot
be consoled by this statement. Which of thefollowing is the most likely
diagnosis?
+A. Body dysmorphic disorder
B. Delusional disorder
C. Obsessive-compulsive disorder
D. Somatization disorder

E. Hypochondriasis
36. A 72-year-old retired English professor with a long history of
hypertension has been having difficulties with tasks he used to find easy
and enjoyable, such as crossword puzzles and letter writing, because he
cannot remember the correct words and his handwriting has
deteriorated. He has also been having difficulties remembering the
events of previous days and he moves and thinks at a slower pace.
Subsequently, he develops a mild right facial hemiparesis and slurred
speech. Your diagnosis is:
A. Wernicke-Korsakoff syndrome
B. Huntingtons disease
C. Alzheimers disease
+D. Vascular dementia
E. Creutzfeldt-Jakob disease
37. Which of the following is a mood stabilizer?
+A. Lamotrigine (Anticonvulsants)
B. Haloperidol

38. Which of the following psychotherapies has best evidence to be


effective in OCD treatment?
A. Biofeedback
+B. Exposure and response prevention
C. Systemic desensitization
39. Which of the following medications could be used in the treatment
of erectile dysfunction?
+A. Sildenafil
40. A case about "premenstrual dysphoric disorder".
41. A case about "narcolepsy".

42. A case about "conduct disorder".


43. A difference between typical and atypical antipsychotics.
44. A case about "conversion disorder".
45. A case about "somatization disorder".

"


."

Sama (2014) Psychiatry Final Questions
The exam was 45 questions, one form.

Q1. A demanding 25-year-old woman begins psychotherapy stating that she


is both desperate and bored. She reports that for the past 6 years she has
experienced periodic anxiety and depression and has made several suicidal
gestures. She also reports a variety of impulsive and self-defeating
behaviors and sexual promiscuity. She wonders if she might be a lesbian,
though most of her sexual experiences have been with men. She has abruptly
terminated two previous attempts at psychotherapy. In both cases she was
angry at the therapist because he was unwilling to prescribe anxiolytic
medications. Which of the following is the most likely diagnosis?
a
b
c
d
e

Antisocial personality disorder


(+)Borderline personality disorder
Histrionic personality disorder
Avoidant personality disorder
Impulse control disorder

Borderline personality disorder : person who suffers from this disorder has
labile interpersonal relationships characterized by instability. This pattern
of interacting with others has persisted for years and is usually closely
related to the person's self-image and early social interactions. The
pattern is present in a variety of settings (e.g., not just at work or home)
and often is accompanied by a similar lability (fluctuating back and forth,
sometimes in a quick manner) in a person's affect, or feelings. Relationships
and the person's affect may often be characterized as being shallow. A
person with this disorder may also exhibit impulsive behaviors and exhibit
at least 5 of the following symptoms:

1) desperate efforts to avoid real or imagined abandonment.


2) a pattern of unstable and intense interpersonal relationships
3) identity disturbance: markedly and persistently unstable self-image or sense of self
4) impulsivity in at least two areas that are potentially self-damaging (e.g., spending, sex,
substance abuse, reckless driving, binge eating)
5) recurrent suicidal behavior, gestures, or threats, or self-mutilating behavior
6) Affect and mood instability
7) chronic feelings of emptiness
8) inappropriate, intense anger or difficulty controlling anger
9) transient, stress-related paranoid ideation or severe dissociative symptoms

Q2. Which of the following features is not true for anorexia nervosa?
a
b
c
d
e

Morbid fear of fatness


It is more prevalent in developing countries
It is more common in females
(+)Patients have body weight more than 85% of the expected
Distorted body image

Dsm 4 criteria anorexia

Body weight at least 15% below normal (less than 85% of the expected)

Have intense fear of gaining weight or becoming fat.

Disturbed body image.

Amenorrhea.

Q3. Which drug of the following antidepressant medications has the lowest
overall rate of sexual dysfunction?
a) Escitalopram SSRI: levo enantiomer of citalopram, similar efficacy

b) Fluoxetine SSRI: longest half life, safe in pregnancy, approved for


use in children
c) Paroxetine SSRI: highly protein bound (several drug interactions),
more anticholinergic effects, short half life -> withdrawal phenomena
d) Amitriptyline TCA (tertiary amines)
e) (+)Bupropion (norepinephrine-dopamine reuptake inhibitors)
-relative lack of sexual side effects as compared to SSRI
-some efficacy in tt of adult ADHD
-s/e include increase risk of seizures and psychosis at high doses
and increase anxiety in some.
-contraindicated in pt with seizure or active eating disorders and
in those currently on an MAOI. (1st aid pg: 186)

Q4. An 8-year-old boy with a family history of tic disorders is referred to


you for an evaluation of behavioral difficulties in school. His teachers report
that he is unable to sit still, constantly fidgets, and is unable to complete
class work because he is so easily distracted. The boy's mother reports that
he has always had a lot of energy. She says that preparing to leave for
school in the morning is extremely difficult because of her son's
disorganization and forgetfulness. She denies that her son produces any
repetitive movements or sounds. Which of the following is the most likely
diagnosis?
a)
b)
c)
d)
e)

Oppositional defiant disorder


(+)Attention deficit hyperactivity disorder
Conduct disorder
Generalized anxiety disorder
Disruptive behavior disorder

ADHD : It is a psychiatric disorder of the neurodevelopmental type in which


there are significant problems of attention, hyperactivity, or acting
impulsively that are not appropriate for a persons age.

Inattention: Six or more symptoms of inattention for children up to age 16, or five or
more for adolescents 17 and older and adults; symptoms of inattention have been present
for at least 6 months, and they are inappropriate for developmental level:
Often fails to give close attention to details or makes careless mistakes in schoolwork, at
work, or with other activities.
Often has trouble holding attention on tasks or play activities.
Often does not seem to listen when spoken to directly.
Often does not follow through on instructions and fails to finish schoolwork, chores, or
duties in the workplace (e.g., loses focus, side-tracked).
Often has trouble organizing tasks and activities.
Often avoids, dislikes, or is reluctant to do tasks that require mental effort over a long
period of time (such as schoolwork or homework).
Often loses things necessary for tasks and activities (e.g. school materials, pencils, books,
tools, wallets, keys, paperwork, eyeglasses, mobile telephones).
Is often easily distracted
Is often forgetful in daily activities.
Hyperactivity and Impulsivity: Six or more symptoms of hyperactivity-impulsivity for children
up to age 16, or five or more for adolescents 17 and older and adults; symptoms of
hyperactivity-impulsivity have been present for at least 6 months to an extent that is
disruptive and inappropriate for the persons developmental level:
Often fidgets with or taps hands or feet, or squirms in seat.
Often leaves seat in situations when remaining seated is expected.
Often runs about or climbs in situations where it is not appropriate (adolescents or adults
may be limited to feeling restless).
Often unable to play or take part in leisure activities quietly.
Is often "on the go" acting as if "driven by a motor".
Often talks excessively.
Often blurts out an answer before a question has been completed.
Often has trouble waiting his/her turn.
Often interrupts or intrudes on others (e.g., butts into conversations or games).

Q5. A 48-year-old alcoholic man with hepatic cirrhosis is admitted for


alcohol detoxification. The attending physician on call instructs you to treat
the patient with routine treatment including Thiamine and Benzodiazepines.
She tells you that, because the patient likely has impaired hepatic
metabolism, you should choose a Benzodiazepine that is metabolized by
conjugation and that doesn't have long-acting metabolites. A reasonable
choice is:
a)
b)
c)
d)
e)

Buspirone
Diazepam
Chlordiazepoxide
Clonazepam
(+)Lorazepam
1st choice : chlordiazepoxide (less euphoric, less addiction)
compared to diazepam
2nd choice : diazepam
Lorazepam, Oxazepam and Temazepam (LOT) if liver damage is
expected.

Q6. A 21-year-old man is brought to the physician by his parents after he


called them from college, terrified that the Mafia was after him. He reports
that he has eaten nothing for the past 6 weeks other than canned beans
because "they are into everything". He is convinced that the Mafia has put
cameras in his dormitory room and that they are watching his every move.
He sometimes hears the voices of two men talking about him when no one is
around. His roommate states that for the past 2 months the patient has
been increasingly withdrawn and suspicious. Which of the following is the
most likely diagnosis?
a)
b)
c)
d)
e)

Schizophrenia
Schizoaffective disorder
(+)Schizophreniform disorder
Bipolar II disorder
Delusional disorder

Schizophreniform Disorder
Diagnosis: Same criteria as Schizophrenia.
One difference, in schizophreniform disorder the symptoms have lasted between 1 and 6
months, whereas in schizophrenia the symptoms must be present for more than 6 months.

DSM-5 Diagnostic Criteria of Schizophrenia


A. Two (or more) of the following, each present for a significant portion of time during a 1
-month period (or less if successfully treated). At least one of these must be (1 ), (2), or
(3):
1.

Delusions

2. Hallucinations
3. Disorganized speech (e.g., frequent derailment or incoherence)
4. Grossly disorganized behavior or catatonic behavior
5. Negative symptoms (such as flattened affect)
B. Social/occupational dysfunction: 1 major areas of functioning (work, interpersonal
relations, self-care) markedly below the level achieved prior to the onset of symptoms
C. Continuous signs of disturbance for 6 mo, including 1 mo of active phase symptoms; may
include prodromal or residual phases
D. Symptoms not due to medical, neurological, or substance-induced disorder

Q7. Which of the following personality disorders is not classified in cluster


B personality disorders (emotional, dramatic)?
a)
b)
c)
d)
e)

Histrionic personality disorder


Borderline personality disorder
Narcissistic personality disorder
Antisocial personality disorder
(+)Schizoid personality disorder
(cluster A: schizoid, schizotypal, paranoid)

Cluster C : avoidant, dependent, obsessive compulsive

Q8. Patients who suffer from depression following myocardial infarction


should be treated with which of the following antidepressants:
a) Tricyclic antidepressants Antiadrenergic properties (cvs side
effects) ; orthostatic hypotension, dizziness, reflex tachy,
arrhythmias, ecg changes (widening QRS, QT, PR interval). Avoid in pt
with preexisting conduction abnormalities or recent MI. (1st aid pg188)
b) Bupropion
c) Trazodone Serotonin receptor antagonist and agonist
Side effects :nausea, dizziness, orthostatic hypotension, cardiac
arrhythmias, sedation or priapism. (1st aid pg186)
d) (+)SSRI
e) Monoamine oxidase inhibitor
s/e : serotonin syndrome (SSRI+MAOI), hypertensive crisis (elevated
BP + cc by headache,sweating nausea and vomiting, photophobia,
autonomic instability,chest pain, arrhythmias and death),orthostatic
hypotension (most common). (1st aid pg 188)

Q9. Which of the following medications is not a mood stabilizer?


a)
b)
c)
d)
e)

Lithium
(+)Donepezil
Carbamazepine
Lamotrigine
Valproic acid

Mood stabilizers : (1st aid pg 194)


1) Lithium
2) Anti-epileptic : carbamazepine and valproic acid (lamotrigine,
oxcarbazepine,gabapentin, pregabalin, topiramate)
Q10. Every 4 or 5 weeks, a usually well-functioning 35-year-old woman
experiences a few days of irritability, tearfulness and unexplained sadness.

During these days, she also feels fatigued and bloated and eats large
quantities of sweets. Which of the following is the most likely diagnosis?
a)
b)
c)
d)
e)

(+)Premenstrual dysphoric disorder


Borderline personality disorder
Dissociative identity disorder
Persistent depressive disorder
Cyclothymia

Criterion A is that in most menstrual cycles during the past year, at least 5 of the following 11
symptoms (including at least 1 of the first 4 listed) were present:
1.Markedly depressed mood, feelings of hopelessness, or self-deprecating thoughts
2.Marked anxiety, tension, feelings of being keyed up or on edge
3.Marked affective lability (eg, feeling suddenly sad or tearful or experiencing increased
sensitivity to rejection)
4.Persistent and marked anger or irritability or increased interpersonal conflicts
5.Decreased interest in usual activities (eg, work, school, friends, and hobbies)
6.Subjective sense of difficulty in concentrating
7.Lethargy, easy fatigability, or marked lack of energy
8.Marked change in appetite, overeating, or specific food cravings
9.Hypersomnia or insomnia
10.A subjective sense of being overwhelmed or out of control
11.Other physical symptoms, such as breast tenderness or swelling, headaches, joint or muscle
pain, a sensation of bloating, or weight gain

Q11. Which of the following is a common adverse effect of


methylphenidate?
a)
b)
c)
d)
e)

(+)Insomnia
Hypotension
Weight gain
Liver toxicity
Tremor

Psychostimulants (dextroamphetamine & amphetamines, methylphenidate,


atomoxetine, modafanil)
Methylphenidate : CNS stimulant. Watch for leucopenia, anemia, increase
LFTs, monitor BP and watch for weight loss and insomnia. (1st aid pg 198)

Q12. A middle-aged man is chronically preoccupied with his health. For


many years he feared that his irregular bowel functions meant he had
cancer. Now he is very worried about having a serious heart disease, despite
his physician's assurance that the occasional "extra beats" he detects when
he checks his pulse are completely benign. Which of the following is the
most likely diagnosis?
a)
b)
c)
d)
e)

Pain disorder
(+)Hypochondriasis
Delusional disorder
Conversion disorder
Somatization disorder

Hypochondriasis (illness anxiety disorder) :


A. Preoccupation with having or acquiring a serious illness.
B. Somatic symptoms are not present or, if present, are only mild in intensity. If another
medical condition is present or there is a high risk for developing a medical condition
(e.g., strong family history is present), the preoccupation is clearly excessive or disproportionate.
C. There is a high level of anxiety about health, and the individual is easily alarmed about
personal health status.
D. The individual performs excessive health-related behaviors (e.g., repeatedly checks
his or her body for signs of illness) or exhibits maladaptive avoidance (e.g., avoids doctor
appointments and hospitals).
E. Illness preoccupation has been present for at least 6 months, but the specific illness
that is feared may change over that period of time.
F. The illness-related preoccupation is not better explained by another mental disorder, such
as somatic symptom disorder, panic disorder, generalized anxiety disorder, body dysmorphic
disorder, obsessive-compulsive disorder, or delusional disorder, somatic type.

Q13. Which one of the following psychotherapies is used mainly for the
treatment of post-traumatic disorder?
a)
b)
c)
d)
e)

Social learning therapy


Exposure and response prevention
(+)Eye movement desensitization and reprocessing therapy
Psychoanalysis
Brief psychodynamic psychotherapy

Eye Movement Desensitization and Reprocessing (EMDR) is a psychotherapy developed by Francine


Shapiro that emphasizes disturbing memories as the cause of psychopathology.[1][2] It is used to alleviate the
symptoms of posttraumatic stress disorder (PTSD).[3]
EMDR is used for individuals who have experienced severe trauma that remains unresolved

Reference:
http://en.wikipedia.org/wiki/Eye_movement_desensitization_and_repr
ocessing

Q14. Which one of the following is not considered as Paraphilia?


a)
b)
c)
d)
e)

Paedophilia
Voyeurism
Fetishism
(+)Dyspareunia
Exhibitionism

Common Paraphilia
o Pedophilia- Sexual gratification from fantasies or behaviors involving
sexual acts with children
o Frotterurism

o Voyerurism - Watching unsuspecting nude individuals (often with


binoculars) in order to obtain sexual pleasure
o Exhibitonism - Exposure of ones genitals to strangers
o Sadism
o Fetishism- Sexual preference for inanimate objects (e.g., shoes or
pantyhose)
o Transvestic fetischism
o Masochism
o Necrophilia
o Telephone scatologia
Reference: slides
Q15. A 37-year-old mildly retarded man with trisomy 21 syndrome has been
increasingly forgetful. He makes frequent mistakes when counting change at
the grocery store where he has worked for several years. In the past, he
used to perform this task without difficulty. He often cannot recall the
names of common objects, and he has started annoying customers with his
intrusive questions. Which of the following is the most likely diagnosis of
this patient?
a)
b)
c)
d)
e)

Wilson disease
Hypothalamic tumor
(+)Alzheimer disease
Thiamine deficiency
Pseudodementia

Alzheimer disease is common in persons with Down syndrome surviving


to middle age. - Reference: slide

Q16. Which of the following is characteristic of Bulimia Nervosa?


a)
b)
c)
d)
e)

Normal paranoid gland


Normal body image
(+)Binge eating followed by vomiting
Somatic delusions
Extreme weight loss due to fasting
Bulimia nervosa involves binge eating combined with behaviors intended to

counteract weight gain, such as vomiting, use of laxatives or diuretics, or excessive


exercise.
Reference :first aid

Q17. Which of the following psychological etiologies of depression and


dysthymia is false?
a)
b)
c)
d)
e)

(+)Existence of positive reinforcement


Loss of a child in adulthood
Negative interpretation of life events
Learned helplessness
Loss of a parent in childhood

Psychosocial/Life Events
Loss of a parent before age 11 is associated with the later development of major
depression. Stable family and social functioning have been shown to be
good prognostic indicators in the course of major depression.
Reference :first aid

Q18. Which of the following is not a cause of delirium in the elderly?


a)
b)
c)
d)
e)

(+)Depressive disorder
Medication toxicity
Urinary tract infection
Electrolyte imbalance
Alcohol withdrawal

ETIOLOGY
Common causes of delirium include:
_ CNS injury or disease
_ Systemic illness
_ Drug abuse/withdrawal
_ Hypoxia
Additional causes of delirium include:
_ Fever
_ Sensory deprivation
_ Medications (anticholinergics, steroids, antipsychotics, antihypertensives,
insulin, etc.)
_ Postop
_ Electrolyte imbalances
Reference :first aid

Q19. A 23-year-old man is brought to the hospital by his parents because


he has not slept, bathed or eaten in the past 3 days. The parents report that
for the past 7 months their son has been acting strangely and "not himself".
They state that he has locking himself in his room, talking to himself and
writing on the walls. 6 weeks prior to the hospital visit, their son became
convinced that a fellow student was stealing his thoughts and making him
unable to learn his school material. On examination, the patient is dirty,
disheveled and aloof. Which of the following is the most likely diagnosis for
this patient?
a)
b)
c)
d)
e)

Schizoaffective disorder
(+)Schizophrenia
Delusional disorder
Bipolar I disorder
Schizoid personality disorder

DSM-IV Criteria
_ Two or more of the following must be present for at least 1 month:
1. Delusions
2. Hallucinations
3. Disorganized speech
4. Grossly disorganized or catatonic behavior
5. Negative symptoms (such as flattened affect)
_ Must cause significant social or occupational functional deterioration

Duration of illness for at least 6 months (including prodromal or residual


periods in which above criteria may not be met)
_ Symptoms not due to medical, neurological, or substance-induced disorder
_

-talking to himself- Hallucinations


-convinced that a fellow student was stealing his thoughts-Delusion
-dirty, disheveled and aloof- significant social or occupational functional
deterioration

-past 7 months- Duration of illness for at least 6 months


Reference :first aid

Q20. Which of the following is not a symptom of obsessive compulsive


disorder?
a)
b)
c)
d)
e)

Obsessions of doubts
(+)Delusions of control
Compulsive hoarding
Obsession of contamination
Obsession of symmetry

COMMON PATTERNS OF OBSESSIONS AND COMPULSIONS


1. Obsessions about contamination followed by excessive washing or
compulsive avoidance of the feared contaminant
2. Obsessions of doubt (forgetting to turn off the stove, lock the door,
etc.) followed by repeated checking to avoid potential danger
3. Obsessions about symmetry followed by compulsively slow performance
of a task (such as eating, showering, etc.)
4. Intrusive thoughts with no compulsion. Thoughts are often sexual or
violent.
Reference :first aid

Q21. A 23-year-old woman arrives at the emergency room complaining that,


out of the blue, she had been seized by an overwhelming fear, associated
with shortness of breath, sweating, trembling and a pounding heart. These

symptoms lasted approximately 20 minutes, and while she was experiencing


them she feared that she was dying or going crazy. The patient has had 4
similar episodes during the past month, and she has been worrying that they
will continue to recur. Which of the following is the most likely diagnosis?
a)
b)
c)
d)
e)

Acute psychotic episode


Social anxiety disorder
Post-traumatic stress disorder
(+)Panic disorder
Generalized anxiety disorder

Panic Disorder
Panic disorder is characterized by the experience of panic attacks accompanied
by persistent fear of having additional attacks.

HIGH-YIELD FACTS

Panic attack criteria:


PANIC
Palpitations
Abdominal distress
Numbness, nausea
Intense fear of death
Choking, chills, chest pain,
sweating, shaking,
shortness of breath
A panic attack may be
mistaken by patient for a
myocardial infarction;
sufferer may seek help in
the emergency department
(ED).
DIAGNOSIS AND DSM-IV CRITERIA
1. Spontaneous recurrent panic attacks (see above) with no obvious precipitant
2. At least one of the attacks has been followed by a minimum of 1
month of the following:
_ Persistent concern about having additional attacks
_ Worry about the implications of the attack (Am I out of control?)
_ A significant change in behavior related to the attacks (avoid situations
that may provoke attack
Reference :first aid

Q22. A 25-year-old woman is diagnosed with schizophrenia. Which of the


following details of her history leads the physician to suspect that her
outcome (prognosis) maybe poor?

a)
b)
c)
d)
e)

She is female
She has no family history of Schizophrenia
She had a sudden onset of her illness
She had a known precipitating factor before she began hearing voices
(+)She had an insidious onset of her illness

Associated with Better Prognosis


_ Later onset
_ Good social support
_ Positive symptoms
_ Mood symptoms
_ Acute onset
_ Female sex
_ Few relapses
_ Good premorbid functioning
Associated with Worse Prognosis
_ Early onset
_ Poor social support
_ Negative symptoms
_ Family history
_ Gradual onset
_ Male sex
_ Many relapses
_ Poor premorbid functioning (social isolation, etc.)
Reference :first aid

Q23. What is the most common cause of Korsakoff syndrome?


a)
b)
c)
d)
e)

Encephalitis
(+)Thiamine deficiency
Hypoxia
Carbon monoxide poisoning
Traumatic brain injury
Long-Term Complications of Alcohol Intake

WernickeKorsakoff syndrome is caused by thiamine (vitamin B 1) deficiency


resulting from the poor diet of alcoholics. Wernickes encephalopathy is acute
and can be reversed with thiamine therapy
Reference :first aid

Q24. Which one of the following is not regarded as psychotherapy?


a) (+)Transcranial magnetic stimulation
b) Family therapy
c) Interpersonal therapy
d) Cognitive behavioral therapy
e) Transactional analysis
http://www.mayoclinic.org/tests-procedures/psychotherapy/basics/what-you-canexpect/prc-20013335
http://www.medicinenet.com/script/main/art.asp?articlekey=25019
*Psychotherapy is a general term for treating mental health problems by talking
with a psychiatrist, psychologist or other mental health provider.
*Types of psychotherapy
-Cognitive behavioral therapy, which helps you identify unhealthy, negative
beliefs and behaviors and replace them with healthy, positive ones
-Dialectical behavior therapy, a type of cognitive behavioral therapy that
teaches behavioral skills to help you handle stress, manage your emotions and
improve your relationships with others
-Acceptance and commitment therapy, which helps you become aware of and
accept your thoughts and feelings and commit to making changes, increasing your
ability to cope with and adjust to situations
-Psychodynamic and psychoanalysis therapies, which focus on increasing your
awareness of unconscious thoughts and behaviors, developing new insights into
your motivations, and resolving conflicts
-Interpersonal psychotherapy, which focuses on addressing problems with your
current relationships with other people to improve your interpersonal skills how
you relate to others, such as family, friends and colleagues
-Supportive psychotherapy, which reinforces your ability to cope with stress and
difficult situations
** Transactional analysis is:
-Group psychotherapy technique in which games are used to expose
childish, manipulative behavior toward life and people to work toward

achieving mature and constructive attitudes.


http://www.businessdictionary.com/definition/transactional-analysis-TA.html
**transcranial magnetic stimulation is:
Transcranial magnetic stimulation: Abbreviated TMS. A non-invasive technique that
consists of a magnetic field emanating from a wire coil held outside the head. The
magnetic field induces an electrical current in nearby regions of the brain. TMS was
originally developed as a diagnostic tool for mapping brain function. It appears
promising as a treatment for some neuropsychiatric conditions, particularly major
depression.
Q25. A 32-year-old woman is brought to the emergency room by the police after she
was found standing in the middle of a busy road, commanding the traffic to stop. In
the emergency room she is agitated and restless with pressured speech and an
affect that alternates between euphoric and irritable. Her father is contacted and
states that she hasn't slept well for the last 10 days and showed increased activity
including excessive involvement in pleasurable activities without thought for
consequences. Which of the following is the most likely diagnosis?
a) Delirium
b) (+)Bipolar disorder, manic
c) Schizophrenia
d) Bipolar disorder, mixed features
e) Cyclothymia
http://www.webmd.com/depression/guide/bipolar-disorder-manic-depression?
page=2#2
*The signs of mania with bipolar disorder include:
-Disconnected and very fast (racing) thoughts
-Grandiose beliefs
-Inappropriate elation or euphoria
-Inappropriate irritability
-Inappropriate social behavior
-Increased sexual desire
-Increased talking speed or volume

-Markedly increased energy


-Poor judgment
-A decreased need for sleep due to high energy
Q26. A 22-year-old woman presents in the out-patient department complaining that
her nose is misshapen. You examine her and find no evidence that it is abnormal.
She pulls out a mirror and looks at herself. "See", she says pointing to her nose,
"don't you see how ugly it is?". She tells you that she cannot continue walking
around in public with this nose. She feels that everyone notices it and has gone to a
number of plastic surgeons to try to get it corrected. None has willing to operate on
her nose to fix it. You ask a colleague to examine the patient, and he draws the
same conclusion as you. The patient has:
a) (+)A body dysmorphic disorder
b) Dissociative disorder
c) Hypochondriasis
d) Borderline personality disorder
e) Social phobiahttp
://www.webmd.com/mental-health/mental-health-body-dysmorphic-disorder#1
*Some of the warning signs that a person may have BDD include:
-Engaging in repetitive and time-consuming behaviors, such as looking in a mirror,
picking at the skin, and trying to hide or cover up the perceived defect
-Constantly asking for reassurance that the defect is not visible or too obvious
-Repeatedly measuring or touching the perceived defect
-Experiencing problems at work or school, or in relationships due to the inability to
stop focusing about the perceived defect
-Feeling self-conscious and not wanting to go out in public, or feeling anxious when
around other people
-Repeatedly consulting with medical specialists, such as plastic surgeons or
dermatologists, to find ways to improve his or her appearance
Q27. Which of the following medications is an antidote for Benzodiazepine overdose
or toxicity?
a) Buspirone

b) Hydroxyzine
c) (+)Flumazenil
d) Naloxone
e) Fluoxetine
http://www.ncbi.nlm.nih.gov/pubmed/8438687
Flumazenil, a specific benzodiazepine antagonist, is useful in reversing the sedation
and respiratory depression that often occur when benzodiazepines are administered
to patients undergoing anesthesia or when patients have taken an intentional
benzodiazepine overdose. Judicious use of flumazenil may provide useful diagnostic
information and may obviate the need for mechanical ventilation and other invasive
supportive measures. Although some controversy exists regarding the possible
precipitation of seizure activity in the setting of mixed tricyclic antidepressantbenzodiazepine overdose, worldwide experience with flumazenil has validated its
safety and efficacy.
Q28. The only tricyclic antidepressant thought to be effective in the treatment of
obsessive-compulsive disorder is:
a) Imipramine
b) (+)Clomipramine
c) Amitriptyline
d) Desipramine
e) Nortriptyline
http://www.patient.co.uk/health/obsessive-compulsive-disorder-leaflet
If SSRIs do not help much, or cannot be taken, then another type of antidepressant
called clomipramine is sometimes used. This is classed as a tricyclic
antidepressant and used to be the main medication treatment for OCD before SSRIs
became available. Occasionally, other medicines that are used to treat mental
health disorders are used.
Clomipramine, a tricyclic antidepressant, has been used for years to treat OCD, but
it may have more side effects than SSRIs.
Antidepressants are used to relieve the obsessive thoughts and subsequent
compulsive behaviors in those who have OCD. By increasing the level of serotonin in
the brain, antidepressants help to regulate the communication between different
parts of the brain.

Q29. A 37-year-old woman comes to the physician with a chief complaint of feeling
sad. The patient states she has anhedonia, anergia, a 4-Kg weight loss in the last 3
weeks and states she "just doesn't care about anything anymore". She also admits
to suicidal ideation without intent or plan. Which of the following is the most likely
diagnosis?
a) (+)Major depression
b) Dysthymic disorder
c) Cyclothymia
d) Mood disorder secondary to a medical condition
e) Normal grief
http://www.webmd.com/depression/guide/major-depression
*symptoms might include:
-Fatigue or loss of energy almost every day
-Feelings of worthlessness or guilt almost every day
-Impaired concentration, indecisiveness
-Insomnia or hypersomnia (excessive sleeping) almost every day
-Markedly diminished interest or pleasure in almost all activities nearly every day
(called anhedonia, this symptom can be indicated by reports from significant others)
-Restlessness or feeling slowed down
-Recurring thoughts of death or suicide
-Significant weight loss or gain (a change of more than 5% of body weight in a
month)
Q30. A 27-year-old man complains of excess sleepiness during the day and having
intense dreams at times when he thinks he is still awake. He notes that he
sometimes has an irresistible urge to sleep and has fallen asleep on multiple
occasions even while driving. He feels refreshed after a daytime nap. Upon
questioning, he reports that he sometimes has "drop" attacks characterized by
sudden weakness in his whole body. He remains alert during the drop attacks but
does fall to the floor. An appropriate treatment intervention is:
a) Antihistamine to help him sleep
b) Administration of Zolpidem at bed time to improve night-time sleep

c) Continuous positive airway pressure at night to improve abnormal breathing


during sleep
d) Valproic acid to treat seizure activity
e) (+)Amphetamine to improve daytime alertness
http://www.medicinenet.com/narcolepsy/page4.htm
*Medications for narcolepsy include:
-Stimulants(methylphenidate (Concerta, Ritalin, others) or various amphetamines)
*Symptoms of narcolepsy include (each of these are discussed in detail):
-excessive daytime sleepiness
-cataplexy
-hypnagogic hallucinations
-sleep paralysis
-disturbed nocturnal sleep
-automatic behavior
-other complaints such as blurred vision,double vision, or droopy eyelids
Q31. A 38-year-old woman with bipolar disorder has been stable on Lithium for the
past 2 years. She comes to her psychiatrist's office in tears after a 2-week history of
a depressed mood, poor concentration, lost appetite and passive suicidal ideation.
Which of the following is the most appropriate next step in the management of this
patient?
a) Stop the Lithium and start an antipsychotic
b) b) Start the patient on a long-acting Benzodiazepine
c) Start the patient on a second mood stabilizer
d) Stop the Lithium and start an antidepressant
e) (+)Start an antidepressant and continue the Lithium
http://www.webmd.com/bipolar-disorder/guide/treating-bipolar-medication
http://www.nytimes.com/health/guides/disease/bipolar-disorder/print.html
*In some patients with bipolar disorder, a mood stabilizer may be all that's needed
to modulate the depressed mood. However, in bipolar patients who do not respond

to one mood stabilizer, another mood stabilizer or an atypical antipsychotic can be


added to the treatment regimen.
*Mood stabilizing drugs are the mainstay for patients with bipolar disorder. They are
defined as drugs that are effective for acute episodes of mania and depression and
that can be used for maintenance. The standard first-line mood stabilizers are
lithium and valproate. Both drugs stimulate the release of the neurotransmitter
glutamate, although they appear to work through different mechanisms. Other
drugs may also be used. Drugs to treat bipolar disorder should be prescribed and
managed by a psychiatrist.
-Lithium .
-Antiseizure Drugs . Valproate (valproic acid) carbamazepine (Tegretol, Carbatrol,
Equetro), oxcarbazepine (Trileptal), and lamotrigine (Lamictal) are the antiseizure
drugs used most often in treating bipolar illness. Other antiseizure drugs used or
investigated for bipolar include gabapentin (Neurontin), zonisamide (Zonegran) and
topiramate (Topamax)..
-Atypical Antipsychotics . They may be used either alone or in combination with
lithium or valproate. Clozapine (Clozaril) was the first of these drugs, but it has not
yet been approved for treatment of bipolar disorder. The newer atypical
antipsychotics include olanzapine (Zyprexa), risperidone (Risperdal), quetiapine
(Seroquel), ziprasidone (Geodon), and ariprazole (Abilify).
-Antidepressants . Antidepressants alone are not recommended, but may be used
with care for depressive symptoms that do not respond to lithium and antiseizure
drugs.. The first choices for antidepressants are bupropion (Wellbutrin) or paroxetine
(Paxil). Alternatives include one of the selective serotonin reuptake inhibitors
(SSRIs), such as fluoxetine
*Such drugs may be used in combination with each other. Additional drugs, such as
conventional antipsychotics, antidepressants, antianxiety drugs, or experimental
drugs are used as necessary.
Q32. Which of the following symptoms differentiate obstructive sleep apnea from
central sleep apnea?
a) Chronic fatigue
b) Unrefreshing sleep
c) Daytime sleepiness
d) (+)Loud snoring
e) Insomnia

http://www.mayoclinic.org/diseases-conditions/central-sleepapnea/basics/symptoms/con-20030485
http://www.mayoclinic.org/diseases-conditions/obstructive-sleepapnea/basics/symptoms/con-20027941
obstructive sleep apnea

central sleep apnea

Excessive daytime sleepiness


Loud snoring

Observed episodes of stopped


breathing or abnormal breathing
patterns during sleep

Observed episodes of breathing


cessation during sleep

Abrupt awakenings accompanied by


shortness of breath

Abrupt awakenings accompanied by


shortness of breath

Shortness of breath that's relieved by


sitting up

Awakening with a dry mouth or sore


throat

Difficulty staying asleep (insomnia)

Awakening with chest pain

Excessive daytime sleepiness


(hypersomnia)

Morning headache

Difficulty concentrating

Difficulty concentrating during the day

Mood changes

Experiencing mood changes, such as


depression or irritability

Morning headaches

Difficulty staying asleep (insomnia)

Snoring

Having high blood pressure

1. Obstructive Sleep Apnea [OSA] which is correlated to snoring or


2. Central Sleep Apnea [SPA] which is correlated to
heart failure.
Q33. A 34-year-old secretary climbs 12 flights of stairs every day to reach her office
because she is terrified by the thought of being trapped in the elevator. She has
never had any traumatic event occur in an elevator, nonetheless, she has been
terrified of them since childhood. Which of the following is the most likely diagnosis?
a) Agoraphobia

b) Performance anxiety
c) Social phobia
d) Generalized anxiety disorder
e) (+)Specific phobia
http://www.mdguidelines.com/phobias-specific
Individuals with specific phobias have unwarranted and intense fears of specific
objects or situations. The most common phobias are of animals (zoophobia), blood
(hemophobia), heights (acrophobia), travel by airplane (aerophobia), being closed in
(claustrophobia), and thunderstorms (keraunophobia). Others include spiders
(arachnophobia), strangers (xenophobia), and crowds (agoraphobia). Phobias may
develop after a firsthand experience of being injured, or after witnessing another
person become injured.
The anxiety produced by exposure to one of these objects or situations may be
a panic attack or more generalized anxiety, but it is always directed at something
specific.

Q34. A 25-year-old woman delivers a healthy baby boy by cesarean section.


She notes over the next week that she has become irritable and feels down.
She worries that her child will die and fantasizes that if the child died; she
would kill herself as well. She reports not being able to sleep, is fatigued and
has lost 3 Kg within 2 weeks. Over the course of the following week, she
begins to investigate how she might commit suicide. Which of the following
is the most likely diagnosis?
a)
b)
c)
d)
e)

Generalized anxiety disorder


Postpartum psychosis
(+)Postpartum depression
Uncomplicated bereavement
Postpartum blues

Explaination: Characterized by depressed affect, anxiety , poor


concentration starting within 4 weeks after delivery. Last 2 weeks to a year
or more

Source:slide seminar

Q35. Which of the following statements concerning autism is false?


a) They have delayed or minimal expressive speech and comprehension
b) Three-forth of autistic children also have intellectual disability
c) Most children with autism present symptoms in the first few years of
life
d) (+)Autism is often the first manifestation of Schizophrenia
e) Children with autism are uncommonly resistant to change

Q36. Which one of the following is not recognized as a symptom of posttraumatic stress disorder (PTSD)?
a)
b)
c)
d)
e)

Hyperarousal
Recurrent nightmares
Flashbacks
(+)Formal thought disorder
Avoiding reminders of the event

->The following criteria of PTSD apply to adults, adolescents,


and children older than 6 years. (DSM-V)
A. Exposure to actual or threatened death, serious
injury, or sexual violence .

B- intrusion symptoms ( 1/5 symptoms needed ):


1. Recurrent, involuntary, and intrusive recollections .
* children > 6 y. may express this symptom in repetitive play.
2. Traumatic nightmares
* Children > 6 y. may have disturbing dreams without content
related to trauma .
3. Dissociative reactions (e.g., flashbacks) in which the
individual feels or acts as if the traumatic event were
recurring.
In children > 6y. May re-enact in play .
4. Intense or prolonged psychological distress after
exposure to traumatic reminders (e.g., windy days after a
hurricane; seeing someone who resembles one's
perpetrator).
5. Marked physiological reactivity after exposure to
trauma-related stimuli .
C. Persistent effortful avoidance of distressing trauma-related stimuli
after the event
D. Negative alterations in cognitions and mood that began
or worsened after the traumatic event ( 2/7 symptoms
needed ) :

1. Inability to remember an important aspect of the


traumatic event (typically due to dissociative amnesia and
not to other factors such as head injury, alcohol, or
drugs).
2. Persistent and exaggerated negative beliefs or
expectations about oneself, or the world (e.g., I am
bad, No one can be trusted, The world is completely
dangerous .
3. Persistent, distorted blame of self or others for causing
the traumatic event or for resulting consequences .
(NEW)
4. Persistent negative emotional state (e.g., fear, horror,
anger, guilt, or shame). (NEW)
5. Markedly diminished interest or participation in (pre-traumatic)
significant activities.
6. Feelings alienated from others (e.g. detachment or
estrangement ).
7. Constricted affect : Persistent inability to experience
positive emotions (e.g., inability to experience
happiness, satisfaction, or loving feelings).
E. Trauma-related alterations in arousal and reactivity

that began or worsened after the traumatic event (2/6


symptoms needed) :
1. Irritable or aggressive behavior (with little or no
provocation) typically expressed as verbal or physical
aggression toward people or objects (e.g., yelling at
people, getting into fights, destroying objects).
2. Reckless or self-destructive behavior such as dangerous
driving, excessive alcohol or drug use . (NEW)
3. Hypervigilance (e.g., following a motor vehicle accident,
being especially sensitive to the threat potentially
caused by cars or trucks)
4. Exaggerated startle response (e.g., jumping markedly in
response to a telephone ringing) .
5. Problems in concentration.
6. Sleep disturbance (e.g., difficulty falling or staying
asleep or restless sleep).
F. Duration of the disturbance (Criteria B, C, D, and E) is
more than 1 month.
SOURCE:Slide seminar
Q37. Which of the following anxiolytics is not associated with the
development of substance use disorder (Dependence)?

a)
b)
c)
d)
e)

Temazepam(benzodiazepine used for treatment of insomnia)


Lorazepam (Intermediate-acting benzodiazepine)
Diazepam (Long-acting benzodiazepine)
(+)Buspirone
Alprazolam(Intermediate-acting benzodiazepine)

-Long-term use of benzodiazepine cause physical dependence and withdrawal


Source: http://en.wikipedia.org/wiki/Benzodiazepine

Q38. A 26-year-old man comes to the psychiatrist because he becomes


extremely anxious in social situations. He is unable to talk to anyone and
experiences sweaty palms and a rapid heartbeat. Which of the following
treatment options will provide the most effective and longest lasting results
for this patient?
a)
b)
c)
d)
e)

Treatment with an anxiolytic


Hypnosis
Psychodynamic psychotherapy
Psychoanalysis
(+)Cognitive-Behavioral therapy

-this is social phobia. The first line treatment for social anxiety
disorder is cognitive behavioral therapy with medications such as
selective serotonin reuptake inhibitors (SSRIs) used only in those who
are not interested in therapy
Source: http://en.wikipedia.org/wiki/Social_anxiety_disorder

Q39. Electroconvulsive therapy (ECT) is contraindicated in which of the


following groups:
a) Patients with osteoporosis

b)
c)
d)
e)

(+)Patients who are poor anesthesia risk


The elderly
Pregnant women
Patients with pre-existing seizure disorder

-Abslute contraindication of ECT: (source:slide seminar dr ali)


1)Brain tumor.
2)TB (Controversial).
3)Increased ICP.
-Relative contraindication :
1)Hypertension (Problem with anesthesia)
2) DM (Problem with anesthesia)
3)Fractures (Problem with ECT)

Q40. A 27-year-old woman has been feeling blue for the past 2 weeks. She
has little energy and has trouble concentrating. She states that 6 weeks ago
she had been feeling very good, with lots of energy and no need for sleep.
She says that this pattern has been occurring for at least the past 3 years
and have no more than 2 symptom-free months in a row though the episodes
have never been so severe that she couldnt work. Which of the following is
the most likely diagnosis?
a)
b)
c)
d)

Bipolar depression
Seasonal affective disorder
Borderline personality disorder
(+)Cyclothymic disorder

e) Major depression, recurrent


-cyclothymic is Alternating periods of hypomania and periods with mild to
moderate depressive symptoms
DSM-IV criteria:Numerous periods with hypomanic symptoms and periods
with depressive symptoms for at least 2 years.The person must never have
been symptom free for > 2 months during those 2 years.
No history of major depressive episode or manic episode
Source;slide seminar

Q41. The use of one of the following mood stabilizers is associated with
increased prevalence of polycystic ovary syndrome:
a)
b)
c)
d)
e)

(+)Valproate
Lamotrigine
Lithium
Carbamazepine
Topiramate

-increased prevalence of polycystic ovary syndrome (PCOS) associated with


valproate(VPA) use has been reported in both women with epilepsy and
women with bipolar disorders.
source: http://www.ncbi.nlm.nih.gov/pubmed/19012099

Q42. A talented 21-year-old violinist's musical career is in jeopardy because


he becomes acutely anxious and distressed whenever he is asked to play in
front of an audience where he might have to do something that will
embarrass him. What is the most likely diagnosis?
a) Specific phobia

b)
c)
d)
e)

Agoraphobia
Panic disorder
Acute stress reaction
(+)Social phobia

->is a fear of social situations in which embarrassment can occur.Social


anxiety only occurs when the patient is subject to the scrutiny of others
(public speaking, oral exam, eating in the cafeteria). May be associated
with panic attacks.
Source:slide seminar

Q43. A 24-year-old man with chronic Schizophrenia under treatment with


antipsychotics is brought to the emergency room after his parents found
him in his bed and were unable to communicate with him. On examination, the
man is confused and disoriented. He has severe muscle rigidity and a
temperature of 39.4 C. his blood pressure is elevated and he has
leukocytosis. Which of the following is the best first step in the
pharmacological treatment of this man?
a)
b)
c)
d)
e)

Lorazepam (Short-acting benzodiazepine)


Haloperidol (typical anti psychotic)
Benztropine
Lithium (treatment for bipolar disorder)
(+)Bromocriptine

-this is case of neuroleptic malignant syndrome(NMS).patient may have


Autonomic instability , Rigidity , myoglubinuria , hyperpyrexia , unstable
vitals.treatment for NMS: Dantrolene , D2 agonist (Bromocriptine)
Source:slide seminar

Q44. A 32-year-old woman is arrested by the police after she is found


crawling through the window of a movie star's home. She states that the
movie star invited her into his home because the two are secretly married
and "it just wouldn't be good for his career if everyone knew". The movie
star denies the two have ever met, but notes that the woman has sent him
hundreds of letters over the past 2 years. The woman has never been in
trouble before and lives an otherwise isolated and unremarkable life. Which
of the following is the most likely diagnosis?
a)
b)
c)
d)
e)

Schizoaffective disorder
(+)Delusional disorder
Cyclothymia
Schizophreniform disorder
Bipolar I disorder

-delusional disorder: 1)Fixed, persistent, non-bizarre untrue belief


2)Lasting more than 1 month (3)Functioning is not impaired (4)No
hallucinations or mood disturbances. Ex: a woman who thinks that she is
married to a celebrity when in fact she is not.
Source:slide seminar

Q45. A 19-year-old man is brought to the emergency room by his parents,


who are worried about his vomiting and profuse diarrhea. On arrival, his
pupils are dilated, his blood pressure is 170/110 mm Hg, and his muscles are
twitching. His parents report that these symptoms started 2 hours earlier.
For the past few days he has been homebound because of a sprained ankle,
and during this time he has been increasingly anxious and restless. He has

been yawning incessantly and has had a runny nose. Which of the following
drugs is this person most likely to be withdrawing from?
a)
b)
c)
d)
e)

Alcohol
(+)Heroin
Benzodiazepine
Cocaine
Phencyclidine

-heroin is opiod. Opioid withdrawal is a syndrome that follows a relative


reduction in heavy and prolonged use.
The syndrome begins within 6-8 hours after the last dose and peaks
between 48 and 72 hours; symptoms disappear in 7-10 days.
The signs and symptoms are the opposite of those of the acute agonist
effects: lacrimation, rhinorrhea, pupillary dilation, piloerection, diaphoresis,
diarrhea,, mild hypertension, tachycardia, fever, and insomnia ,A flulike
syndrome subsequently develops, with complaints, demands, and drug
seeking.
Source:slide seminar

The End

..

S-ar putea să vă placă și